This scale helps evaluate the immediate danger of self-harm in patients in a psychiatric setting when assessing Suicidal intent.
Question 10
In evidence-based hierarchy, which study design is ranked HIGHEST?
A) Case-control
B) Cohort
C) Cross-sectional
D) Randomized controlled trial (RCT)
E) Systematic review
Answer: E. Systematic review
Explanation:
Systematic reviews synthesize results from multiple RCTs or observational studies, applying rigorous methods to minimize bias. Because they integrate all high-quality evidence on a question, they sit at the top of the evidence pyramid. Individual RCTs come next, then cohort and case-control studies, with cross-sectional near the bottom.
Question 11
A meta-analysis assumes fixed effects when:
A) Study heterogeneity is high
B) Effect sizes vary significantly
C) Population and interventions are homogeneous
D) Publication bias is present
E) Sample sizes are small
Answer: C. Population and interventions are homogeneous
Explanation:
A fixed-effect model treats all studies as estimating the same true effect size under identical conditions. That assumption only makes sense when subjects, interventions, and outcomes are sufficiently similar across studies—i.e., minimal heterogeneity. Thus, it's when Population and interventions are homogeneous.
Question 12
Which measure of central tendency is MOST appropriate for skewed data (e.g., income distribution)?
A) Mean
B) Median
C) Mode
D) Range
E) Standard deviation
Answer: B. Median
Explanation:
The Median marks the 50th percentile and isn’t influenced by extreme values. In skewed distributions—where a few very large or small observations would drag the mean away from the center—the median remains the best single-value summary of the “typical” case.
Question 13
Negative skew in a dataset is indicated by:
A) Mean > Mode
B) Mean < Mode
C) Median = Mean
D) Long right tail
E) Symmetrical distribution
Answer: B. Mean < Mode
Explanation:
Negative skew means the left tail extends farther than the right. That pulls the arithmetic mean below the mode (hence Mean < Mode), since extreme low values drag down the average more than the most common value.
Question 14
A physician decides to work at a rural clinic instead of a city hospital. This decision BEST illustrates:
A) Sunk cost fallacy
B) Opportunity cost
C) Marginal utility
D) Diminishing returns
E) Cost-benefit analysis
Answer: B. Opportunity cost
Explanation:
Opportunity cost is the value of the next-best alternative you give up. By choosing the rural clinic, the physician sacrifices whatever salary, caseload, or career development they could have had in the city. Recognizing that forgone benefit is exactly the concept of Opportunity cost.
Question 15
A researcher validates a new depression screening tool against the PHQ-9. Which type of validity are they assessing?
A) Predictive validity
B) Concurrent validity
C) Content validity
D) Construct validity
E) Criterion validity
Answer: B. Concurrent validity
Explanation:
Concurrent validity checks how well a new measure correlates with an established “gold standard” when both are administered at the same time. Here, comparing the new tool’s scores directly against PHQ-9 results in the same sample assesses Concurrent validity.
Question 2
A clinician adjusts the CAGE cut-off score from 2 to 3. What is the most likely effect on its diagnostic metrics?
A) Increased sensitivity, decreased specificity
B) Decreased sensitivity, increased specificity
C) Increased positive predictive value, unchanged negative predictive value
Answer: B. Decreased sensitivity, increased specificity
Explanation:
The CAGE questionnaire gives one point for each “yes” answer across its four items. By raising the threshold from two affirmative responses to three, you make it harder to screen positive. As a result, fewer true cases of alcohol misuse will be flagged (sensitivity falls), but you also reduce the number of false positives (specificity rises). In other words, you miss more people who do have a problem but you’re less likely to wrongly label someone as disordered. This results in Decreased sensitivity, increased specificity.
Question 3
A researcher conducts a study to evaluate a new depression screening tool. They report a P-value of 0.04. Which statement accurately interprets this result?
A) There is a 4% probability the null hypothesis is true.
B) The tool’s diagnostic accuracy is 96% reliable.
C) There is a 4% chance of falsely rejecting the null hypothesis.
D) The effect size is 0.04.
E) The study has 96% power.
Answer: C. There is a 4% chance of falsely rejecting the null hypothesis.
Explanation:
A P-value of 0.04 means that if in reality there were no effect (the null hypothesis is true), you would obtain data at least as extreme as observed in only 4 out of 100 repeated experiments. It does not tell you the probability that the null hypothesis itself is true, nor does it speak to accuracy, effect size, or power directly. Therefore, there is a 4% chance of falsely rejecting the null hypothesis (Type I error rate if the null is true).
Question 4
A study finds no difference in depression scores between two therapies but reports significant improvements in sleep as a secondary outcome. This MOST likely indicates:
A) Reporting bias
B) Confounding bias
C) Selection bias
D) Observer bias
E) Random error
Answer: A. Reporting bias
Explanation:
When researchers highlight favorable secondary outcomes (like sleep) despite a negative primary endpoint (depression scores), they risk giving a skewed picture of benefit. This selective emphasis—reporting only the “good news”—is classic Reporting bias. Other biases (confounding, selection, observer) involve design or measurement flaws, not selective presentation of results.
Question 5
In optimizing the CAGE questionnaire’s likelihood ratio for detecting harmful alcohol use, which cutoff score provides the highest positive likelihood ratio (LR+)?
A) 0 (any affirmative response)
B) 1
C) 2
D) 3
E) 4
Answer: C. 2
Explanation:
The positive likelihood ratio (LR+) equals sensitivity ÷ (1 – specificity). At a cutoff of two “yes” answers, the CAGE typically yields about 72% sensitivity and 75% specificity, giving an LR+ around 2.9. Lower cutoffs boost sensitivity but harm specificity, reducing LR+. Higher cutoffs improve specificity further but sacrifice too much sensitivity, again lowering LR+. Thus, a cutoff of 2 strikes the best overall balance for ruling in harmful use.
Question 6
Which statistical graph is used in meta-analyses to visually assess heterogeneity across studies?
A) Forest plot
B) Galbraith plot
C) Funnel plot
D) Scatterplot
E) Box plot
Answer: B. Galbraith plot
Explanation:
A Galbraith plot (also called a radial plot) graphs each study’s standardized effect size against its precision (the inverse of its standard error). Outlier studies will stand apart from the main cluster, flagging heterogeneity. While Forest plots summarize individual results and Funnel plots detect publication bias, only the Galbraith plot directly highlights between-study variability.
Question 7
What is the PRIMARY purpose of a Galbraith plot in systematic reviews?
A) Compare treatment effects
B) Assess publication bias
C) Evaluate heterogeneity
D) Visualize publication dates
E) Calculate odds ratios
Answer: C. Evaluate heterogeneity
Explanation:
Galbraith plots are specifically designed to flag heterogeneity by showing which studies deviate most from the average effect size once you adjust for precision. They help reviewers decide whether a fixed-effect or random-effects model is more appropriate, thus their primary purpose is to Evaluate heterogeneity.
Question 8
A patient answers “No” to 3 CAGE questions. What is the BEST interpretation?
A) High specificity for alcohol dependence
B) Moderate sensitivity for hazardous drinking
C) Low probability of alcohol use disorder
D) Confirmatory evidence of sobriety
E) Indication for liver function testing
Answer: C. Low probability of alcohol use disorder
Explanation:
A total score of zero on the CAGE (three “No” answers plus one remaining, assuming the question implies only one "Yes" or zero "Yes" answers if 3 are "No") yields high specificity—meaning few healthy people will test positive—and so a negative result effectively rules out disorder. It doesn’t absolutely confirm sobriety, but it does make alcohol use disorder unlikely, indicating a Low probability of alcohol use disorder.
Question 17
A study compares pain levels in three groups: medication A, B, and C. Which statistical test is MOST appropriate?
A) Unpaired t-test
B) Mann-Whitney U test
C) Kruskal-Wallis test
D) ANOVA
E) Chi-square test
Answer: C. Kruskal-Wallis test
Explanation:
The Kruskal-Wallis test is the non-parametric equivalent of one-way ANOVA and is used to compare three or more independent groups when the outcome (e.g., pain scores) may not follow a normal distribution. It ranks all observations and tests whether the distributions of ranks differ across groups, making it ideal for skewed or ordinal data without assuming equal variances or normality.
Question 18
A researcher compares baseline characteristics between treatment and control groups. Which test adjusts for confounding variables like age and sex?
A) Unpaired t-test
B) ANCOVA
C) Chi-square test
D) Fisher’s exact test
E) Wilcoxon signed-rank test
Answer: B. ANCOVA
Explanation:
Analysis of covariance (ANCOVA) combines ANOVA and regression: it compares means of a continuous outcome across groups while statistically controlling for one or more continuous or categorical covariates (e.g., age, sex). By including confounders in the model, ANCOVA isolates the effect of the main group factor on the outcome, reducing bias from imbalanced baseline characteristics.
Question 19
A psychiatrist compares pre- and post-intervention depression scores in two independent groups. Which test is BEST?
A) Paired t-test
B) Unpaired t-test
C) Mann-Whitney U test
D) Wilcoxon signed-rank test
E) Repeated-measures ANOVA
Answer: B. Unpaired t-test
Explanation:
When comparing the means of a continuous outcome between two independent groups at a single time point (e.g., post-intervention scores, or change scores if pre-subtracted), the two-sample (Unpaired t-test) is appropriate, provided the data are approximately normally distributed and variances are similar. It tests whether the difference in group means is statistically significant.
Question 20
A 2×2 contingency table shows a rare outcome (<5% frequency). Which test is MOST appropriate?
A) Chi-square test
B) Fisher’s exact test
C) McNemar’s test
D) Cochran’s Q test
E) Mantel-Haenszel test
Answer: B. Fisher’s exact test
Explanation:
In a 2×2 contingency table, if the expected frequency in any cell is less than 5 (common when outcomes are rare), the chi-square approximation becomes unreliable. Fisher’s exact test computes the exact probability of observing the data under the null hypothesis and is the standard method for small sample or low-frequency tables.
Question 21
A table reports adjusted odds ratio (OR = 1.5) and unadjusted OR = 2.0. Why is the adjusted OR lower?
A) Confounding by age
B) Smaller sample size
C) Type I error
D) Measurement bias
E) Random variation
Answer: A. Confounding by age
Explanation:
An unadjusted odds ratio may overestimate an association if a confounding variable (e.g., age) is unevenly distributed between groups and is associated with both the exposure and outcome. By including age as a covariate in a multivariable logistic regression model, the adjusted OR accounts for its effect, often attenuating the estimate when age was positively associated with both exposure and outcome. This is described as Confounding by age.
Question 27
Which study design best examines the association between lithium levels and depression severity?
A) Case-control
B) Cohort
C) Ecological
D) Cross-sectional
E) Meta-analysis
Answer: B. Cohort
Explanation:
A prospective cohort study enrolls participants based on exposure status (e.g., different lithium levels) and follows them over time to measure subsequent outcomes (depression severity). This design establishes temporal sequence, reduces recall bias, and allows direct measurement of incidence and risk related to the exposure. Thus, a Cohort study is most appropriate.
Question 28
Which study design is initially most appropriate to evaluate side effects of a new drug in 3 exposed individuals?
A) Case series
B) Case-control
C) N-of-1 trial
D) Cohort
E) Randomized controlled trial
Answer: A. Case series
Explanation:
A Case series describes and analyzes the clinical course of a small group of patients who share a specific exposure (here, 3 individuals on the new drug). It is a descriptive study useful for early identification of potential side effects before larger, more formal studies are undertaken.
Question 32
Two radiologists interpreting MRI results are assessing:
A. Concurrent validity
B. Content validity
C. Inter-rater reliability
D. Predictive validity
E. Internal consistency
Answer: C. Inter-rater reliability
Explanation (quick-revision points):
• Reliability deals with consistency; inter-rater reliability is consistency between observers rating the same cases.
• In psychometrics it is commonly quantified with κ (kappa) for categorical ratings or the intraclass correlation coefficient for continuous data.
A suicide risk tool’s scores correlate with future suicide attempts. This demonstrates:
A. Predictive validity
B. Concurrent validity
C. Content validity
D. Construct validity
E. Face validity
Answer: A. Predictive validity
Explanation:
• Predictive validity asks: does today’s score forecast a later, clinically meaningful outcome?
• Criterion validity has two flavours – concurrent (same time-point) and predictive (future event).
• High scores that accurately anticipate later self-harm indicate sound predictive validity.
Question 34
An ROC curve with an AUC of 0.86 and sensitivity of 100 % indicates:
A) Poor specificity
B) Optimal screening accuracy
C) High false-positive rate
D) Inadequate discriminative ability
E) Superior positive predictive value
Answer: B. Optimal screening accuracy
Explanation:
• AUC > 0.8 = excellent overall discrimination; a value of 0.86 is well inside this zone.
• Sensitivity 100 % means no diseased cases are missed. Where the curve hugs the top-left corner the test achieves its best balance.
• Specificity can still vary at different cut-offs, but the combination of very high sensitivity and high AUC reflects near-ideal screening performance, indicating Optimal screening accuracy.
MRCPsych Paper B - 21 May 2025
MRCPsych Paper B - 21 May 2025
Question 1
Dundrum Crisis Scale primarily assesses:
A) Suicidal intent
B) Risk of violence
C) Functional impairment
D) Insight
E) Medication adherence
Answer: A. Suicidal intent
Explanation:
This scale helps evaluate the immediate danger of self-harm in patients in a psychiatric setting when assessing Suicidal intent.
Question 10
In evidence-based hierarchy, which study design is ranked HIGHEST?
A) Case-control
B) Cohort
C) Cross-sectional
D) Randomized controlled trial (RCT)
E) Systematic review
Answer: E. Systematic review
Explanation:
Systematic reviews synthesize results from multiple RCTs or observational studies, applying rigorous methods to minimize bias. Because they integrate all high-quality evidence on a question, they sit at the top of the evidence pyramid. Individual RCTs come next, then cohort and case-control studies, with cross-sectional near the bottom.
Question 11
A meta-analysis assumes fixed effects when:
A) Study heterogeneity is high
B) Effect sizes vary significantly
C) Population and interventions are homogeneous
D) Publication bias is present
E) Sample sizes are small
Answer: C. Population and interventions are homogeneous
Explanation:
A fixed-effect model treats all studies as estimating the same true effect size under identical conditions. That assumption only makes sense when subjects, interventions, and outcomes are sufficiently similar across studies—i.e., minimal heterogeneity. Thus, it's when Population and interventions are homogeneous.
Question 12
Which measure of central tendency is MOST appropriate for skewed data (e.g., income distribution)?
A) Mean
B) Median
C) Mode
D) Range
E) Standard deviation
Answer: B. Median
Explanation:
The Median marks the 50th percentile and isn’t influenced by extreme values. In skewed distributions—where a few very large or small observations would drag the mean away from the center—the median remains the best single-value summary of the “typical” case.
Question 13
Negative skew in a dataset is indicated by:
A) Mean > Mode
B) Mean < Mode
C) Median = Mean
D) Long right tail
E) Symmetrical distribution
Answer: B. Mean < Mode
Explanation:
Negative skew means the left tail extends farther than the right. That pulls the arithmetic mean below the mode (hence Mean < Mode), since extreme low values drag down the average more than the most common value.
Question 14
A physician decides to work at a rural clinic instead of a city hospital. This decision BEST illustrates:
A) Sunk cost fallacy
B) Opportunity cost
C) Marginal utility
D) Diminishing returns
E) Cost-benefit analysis
Answer: B. Opportunity cost
Explanation:
Opportunity cost is the value of the next-best alternative you give up. By choosing the rural clinic, the physician sacrifices whatever salary, caseload, or career development they could have had in the city. Recognizing that forgone benefit is exactly the concept of Opportunity cost.
Question 15
A researcher validates a new depression screening tool against the PHQ-9. Which type of validity are they assessing?
A) Predictive validity
B) Concurrent validity
C) Content validity
D) Construct validity
E) Criterion validity
Answer: B. Concurrent validity
Explanation:
Concurrent validity checks how well a new measure correlates with an established “gold standard” when both are administered at the same time. Here, comparing the new tool’s scores directly against PHQ-9 results in the same sample assesses Concurrent validity.
Question 2
A clinician adjusts the CAGE cut-off score from 2 to 3. What is the most likely effect on its diagnostic metrics?
A) Increased sensitivity, decreased specificity
B) Decreased sensitivity, increased specificity
C) Increased positive predictive value, unchanged negative predictive value
Answer: B. Decreased sensitivity, increased specificity
Explanation:
The CAGE questionnaire gives one point for each “yes” answer across its four items. By raising the threshold from two affirmative responses to three, you make it harder to screen positive. As a result, fewer true cases of alcohol misuse will be flagged (sensitivity falls), but you also reduce the number of false positives (specificity rises). In other words, you miss more people who do have a problem but you’re less likely to wrongly label someone as disordered. This results in Decreased sensitivity, increased specificity.
Question 3
A researcher conducts a study to evaluate a new depression screening tool. They report a P-value of 0.04. Which statement accurately interprets this result?
A) There is a 4% probability the null hypothesis is true.
B) The tool’s diagnostic accuracy is 96% reliable.
C) There is a 4% chance of falsely rejecting the null hypothesis.
D) The effect size is 0.04.
E) The study has 96% power.
Answer: C. There is a 4% chance of falsely rejecting the null hypothesis.
Explanation:
A P-value of 0.04 means that if in reality there were no effect (the null hypothesis is true), you would obtain data at least as extreme as observed in only 4 out of 100 repeated experiments. It does not tell you the probability that the null hypothesis itself is true, nor does it speak to accuracy, effect size, or power directly. Therefore, there is a 4% chance of falsely rejecting the null hypothesis (Type I error rate if the null is true).
Question 4
A study finds no difference in depression scores between two therapies but reports significant improvements in sleep as a secondary outcome. This MOST likely indicates:
A) Reporting bias
B) Confounding bias
C) Selection bias
D) Observer bias
E) Random error
Answer: A. Reporting bias
Explanation:
When researchers highlight favorable secondary outcomes (like sleep) despite a negative primary endpoint (depression scores), they risk giving a skewed picture of benefit. This selective emphasis—reporting only the “good news”—is classic Reporting bias. Other biases (confounding, selection, observer) involve design or measurement flaws, not selective presentation of results.
Question 5
In optimizing the CAGE questionnaire’s likelihood ratio for detecting harmful alcohol use, which cutoff score provides the highest positive likelihood ratio (LR+)?
A) 0 (any affirmative response)
B) 1
C) 2
D) 3
E) 4
Answer: C. 2
Explanation:
The positive likelihood ratio (LR+) equals sensitivity ÷ (1 – specificity). At a cutoff of two “yes” answers, the CAGE typically yields about 72% sensitivity and 75% specificity, giving an LR+ around 2.9. Lower cutoffs boost sensitivity but harm specificity, reducing LR+. Higher cutoffs improve specificity further but sacrifice too much sensitivity, again lowering LR+. Thus, a cutoff of 2 strikes the best overall balance for ruling in harmful use.
Question 6
Which statistical graph is used in meta-analyses to visually assess heterogeneity across studies?
A) Forest plot
B) Galbraith plot
C) Funnel plot
D) Scatterplot
E) Box plot
Answer: B. Galbraith plot
Explanation:
A Galbraith plot (also called a radial plot) graphs each study’s standardized effect size against its precision (the inverse of its standard error). Outlier studies will stand apart from the main cluster, flagging heterogeneity. While Forest plots summarize individual results and Funnel plots detect publication bias, only the Galbraith plot directly highlights between-study variability.
Question 7
What is the PRIMARY purpose of a Galbraith plot in systematic reviews?
A) Compare treatment effects
B) Assess publication bias
C) Evaluate heterogeneity
D) Visualize publication dates
E) Calculate odds ratios
Answer: C. Evaluate heterogeneity
Explanation:
Galbraith plots are specifically designed to flag heterogeneity by showing which studies deviate most from the average effect size once you adjust for precision. They help reviewers decide whether a fixed-effect or random-effects model is more appropriate, thus their primary purpose is to Evaluate heterogeneity.
Question 8
A patient answers “No” to 3 CAGE questions. What is the BEST interpretation?
A) High specificity for alcohol dependence
B) Moderate sensitivity for hazardous drinking
C) Low probability of alcohol use disorder
D) Confirmatory evidence of sobriety
E) Indication for liver function testing
Answer: C. Low probability of alcohol use disorder
Explanation:
A total score of zero on the CAGE (three “No” answers plus one remaining, assuming the question implies only one "Yes" or zero "Yes" answers if 3 are "No") yields high specificity—meaning few healthy people will test positive—and so a negative result effectively rules out disorder. It doesn’t absolutely confirm sobriety, but it does make alcohol use disorder unlikely, indicating a Low probability of alcohol use disorder.
Question 17
A study compares pain levels in three groups: medication A, B, and C. Which statistical test is MOST appropriate?
A) Unpaired t-test
B) Mann-Whitney U test
C) Kruskal-Wallis test
D) ANOVA
E) Chi-square test
Answer: C. Kruskal-Wallis test
Explanation:
The Kruskal-Wallis test is the non-parametric equivalent of one-way ANOVA and is used to compare three or more independent groups when the outcome (e.g., pain scores) may not follow a normal distribution. It ranks all observations and tests whether the distributions of ranks differ across groups, making it ideal for skewed or ordinal data without assuming equal variances or normality.
Question 18
A researcher compares baseline characteristics between treatment and control groups. Which test adjusts for confounding variables like age and sex?
A) Unpaired t-test
B) ANCOVA
C) Chi-square test
D) Fisher’s exact test
E) Wilcoxon signed-rank test
Answer: B. ANCOVA
Explanation:
Analysis of covariance (ANCOVA) combines ANOVA and regression: it compares means of a continuous outcome across groups while statistically controlling for one or more continuous or categorical covariates (e.g., age, sex). By including confounders in the model, ANCOVA isolates the effect of the main group factor on the outcome, reducing bias from imbalanced baseline characteristics.
Question 19
A psychiatrist compares pre- and post-intervention depression scores in two independent groups. Which test is BEST?
A) Paired t-test
B) Unpaired t-test
C) Mann-Whitney U test
D) Wilcoxon signed-rank test
E) Repeated-measures ANOVA
Answer: B. Unpaired t-test
Explanation:
When comparing the means of a continuous outcome between two independent groups at a single time point (e.g., post-intervention scores, or change scores if pre-subtracted), the two-sample (Unpaired t-test) is appropriate, provided the data are approximately normally distributed and variances are similar. It tests whether the difference in group means is statistically significant.
Question 20
A 2×2 contingency table shows a rare outcome (<5% frequency). Which test is MOST appropriate?
A) Chi-square test
B) Fisher’s exact test
C) McNemar’s test
D) Cochran’s Q test
E) Mantel-Haenszel test
Answer: B. Fisher’s exact test
Explanation:
In a 2×2 contingency table, if the expected frequency in any cell is less than 5 (common when outcomes are rare), the chi-square approximation becomes unreliable. Fisher’s exact test computes the exact probability of observing the data under the null hypothesis and is the standard method for small sample or low-frequency tables.
Question 21
A table reports adjusted odds ratio (OR = 1.5) and unadjusted OR = 2.0. Why is the adjusted OR lower?
A) Confounding by age
B) Smaller sample size
C) Type I error
D) Measurement bias
E) Random variation
Answer: A. Confounding by age
Explanation:
An unadjusted odds ratio may overestimate an association if a confounding variable (e.g., age) is unevenly distributed between groups and is associated with both the exposure and outcome. By including age as a covariate in a multivariable logistic regression model, the adjusted OR accounts for its effect, often attenuating the estimate when age was positively associated with both exposure and outcome. This is described as Confounding by age.
Question 27
Which study design best examines the association between lithium levels and depression severity?
A) Case-control
B) Cohort
C) Ecological
D) Cross-sectional
E) Meta-analysis
Answer: B. Cohort
Explanation:
A prospective cohort study enrolls participants based on exposure status (e.g., different lithium levels) and follows them over time to measure subsequent outcomes (depression severity). This design establishes temporal sequence, reduces recall bias, and allows direct measurement of incidence and risk related to the exposure. Thus, a Cohort study is most appropriate.
Question 28
Which study design is initially most appropriate to evaluate side effects of a new drug in 3 exposed individuals?
A) Case series
B) Case-control
C) N-of-1 trial
D) Cohort
E) Randomized controlled trial
Answer: A. Case series
Explanation:
A Case series describes and analyzes the clinical course of a small group of patients who share a specific exposure (here, 3 individuals on the new drug). It is a descriptive study useful for early identification of potential side effects before larger, more formal studies are undertaken.
Question 32
Two radiologists interpreting MRI results are assessing:
A. Concurrent validity
B. Content validity
C. Inter-rater reliability
D. Predictive validity
E. Internal consistency
Answer: C. Inter-rater reliability
Explanation (quick-revision points):
• Reliability deals with consistency; inter-rater reliability is consistency between observers rating the same cases.
• In psychometrics it is commonly quantified with κ (kappa) for categorical ratings or the intraclass correlation coefficient for continuous data.
A suicide risk tool’s scores correlate with future suicide attempts. This demonstrates:
A. Predictive validity
B. Concurrent validity
C. Content validity
D. Construct validity
E. Face validity
Answer: A. Predictive validity
Explanation:
• Predictive validity asks: does today’s score forecast a later, clinically meaningful outcome?
• Criterion validity has two flavours – concurrent (same time-point) and predictive (future event).
• High scores that accurately anticipate later self-harm indicate sound predictive validity.
Question 34
An ROC curve with an AUC of 0.86 and sensitivity of 100 % indicates:
A) Poor specificity
B) Optimal screening accuracy
C) High false-positive rate
D) Inadequate discriminative ability
E) Superior positive predictive value
Answer: B. Optimal screening accuracy
Explanation:
• AUC > 0.8 = excellent overall discrimination; a value of 0.86 is well inside this zone.
• Sensitivity 100 % means no diseased cases are missed. Where the curve hugs the top-left corner the test achieves its best balance.
• Specificity can still vary at different cut-offs, but the combination of very high sensitivity and high AUC reflects near-ideal screening performance, indicating Optimal screening accuracy.
Question 61
A 38-year-old man with a 10-year history of schizophrenia presents with galactorrhoea and erectile dysfunction. His current regimen includes procyclidine 20 mg daily and flupentixol depot. What is the next best intervention?
A. Add amisulpride
B. Discontinue procyclidine
C. Switch to aripiprazole
D. Reduce flupentixol dose
E. Add propranolol
Answer: C. Switch to aripiprazole
Explanation:Dopamine D2 blockade from flupentixol causes hyperprolactinaemia, leading to galactorrhoea and sexual dysfunction. Aripiprazole, a partial D2 agonist, lowers prolactin levels while preserving antipsychotic efficacy, making it the intervention of choice for antipsychotic-induced hyperprolactinaemia.
Question 63
A 50-year-old man with treatment-resistant schizophrenia and type 2 diabetes mellitus develops severe orthostatic hypotension. His current antipsychotic is olanzapine. Which agent is preferable for switching?
A. Aripiprazole
B. Quetiapine
C. Haloperidol
D. Clozapine
E. Risperidone
Answer: A. Aripiprazole
Explanation:Orthostatic hypotension is primarily mediated by α1-adrenergic receptor antagonism. Aripiprazole has minimal α1-blocking activity and thus carries a lower risk of orthostatic hypotension compared to olanzapine and other antipsychotics with stronger α1 antagonism.
Question 68
A 16-year-old girl restricts her diet to 800 kcal/day, has a BMI of 17, and believes she is “fat,” but her menstrual cycle remains regular. What is the MOST likely diagnosis?
Explanation: Under ICD-10, “anorexia nervosa” requires weight loss plus amenorrhea in females, whereas “atypical anorexia nervosa” meets the psychological and behavioural criteria with significant weight loss but without the menstrual disturbance. This patient has the characteristic low BMI (<17.5), intense fear of weight gain, and body-image disturbance, yet continues to menstruate, fitting atypical anorexia nervosa.
Question 69
A 22-year-old woman with anorexia nervosa is found to have severe hypokalemia (K⁺ 2.8 mmol/L). Which complication is MOST concerning?
A) Respiratory alkalosis
B) Cardiac arrhythmias
C) Metabolic acidosis
D) Muscle weakness
E) Rhabdomyolysis
Answer: B) Cardiac arrhythmias
Explanation: Severe hypokalemia predisposes to life-threatening ventricular arrhythmias (including torsades de pointes) by increasing myocardial excitability. While muscle weakness and metabolic alkalosis can also occur, the immediate risk is cardiac dysrhythmia.
Question 71
Prader-Willi syndrome follows which inheritance pattern?
A) Autosomal recessive
B) Maternal uniparental disomy (UPD) of chromosome 15
C) Paternal deletion on chromosome 15q11–q13
D) X-linked dominant
E) Sporadic mutation
Answer: C) Paternal deletion on chromosome 15q11–q13
Explanation: About 70% of Prader-Willi cases arise from deletion or inexpression of paternal genes on 15q11–q13; maternal UPD accounts for most of the remainder.
Question 73
Genomic imprinting errors explain the etiology of:
A) Fragile X syndrome and Prader-Willi syndrome
B) Angelman syndrome and Prader-Willi syndrome
C) Rett syndrome and Cri-du-chat syndrome
D) Down syndrome and Turner syndrome
E) Klinefelter syndrome and XYY syndrome
Answer: B) Angelman syndrome (maternal deletion) and Prader-Willi syndrome (paternal deletion)
Explanation: Both disorders involve the same chromosomal region (15q11–q13) but differ by parent-of-origin: Angelman syndrome results from maternal deletion or mutation, whereas Prader-Willi syndrome stems from paternal deletion.
Question 75
A newborn weighs 2 kg, has microcephaly, challenging behaviour, and poor memory. Maternal history reveals chronic alcohol use. Which condition is most likely?
A) Fetal Alcohol Spectrum Disorder (FASD)
B) Cocaine exposure
C) Maternal smoking
D) Valproate teratogenicity
E) Lead poisoning
Answer: A) Fetal Alcohol Spectrum Disorder (FASD)
Explanation:FASD encompasses growth restriction (low birth weight), microcephaly, neurocognitive deficits (including memory impairments), and behavioural problems secondary to in utero alcohol exposure.
Question 76
True statement regarding Down syndrome inheritance:
A) Robertsonian translocation is sporadic
B) Trisomy 21 is maternally inherited
C) Mosaicism arises from paternal nondisjunction
D) Robertsonian translocation can be inherited
E) Maternal age is irrelevant
Answer: D) Robertsonian translocation can be inherited
Explanation: In about 95% of Down syndrome cases an extra chromosome 21 arises from maternal nondisjunction, but roughly 4% result from a Robertsonian translocation. Unlike full trisomy, these translocations can be carried in a balanced form by a parent and passed on to children, giving a recurrence risk of around 10% for future pregnancies.
Question 78
ASD heritability is approximately:
A) 40%
B) 60%
C) 90%
D) 20%
E) 50%
Answer: C) 90%
Explanation: Twin and family studies consistently show that genetic factors account for about 90% of the liability to autism spectrum disorders, with environmental influences explaining the remainder.
Question 79
A 19-year-old with self-harm scars and hyperuricemia is MOST likely to have:
A) Lesch-Nyhan syndrome
B) Prader-Willi syndrome
C) Fragile X syndrome
D) Smith-Magenis syndrome
E) DiGeorge syndrome
Answer: A) Lesch-Nyhan syndrome
Explanation:Lesch-Nyhan is caused by an X-linked HGPRT enzyme deficiency leading to uric acid overproduction (hyperuricemia) and severe, compulsive self-mutilation (biting of lips and fingers), often with choreoathetoid movements.
Question 80
A child with macrocephaly, hypotonia, and avoidance of eye contact is MOST suggestive of:
A) Fragile X syndrome
B) Prader-Willi syndrome
C) Autism spectrum disorder
D) Williams syndrome
E) Rett syndrome
Answer: A) Fragile X syndrome
Explanation:Fragile X arises from a CGG expansion in the FMR1 gene and presents with an enlarged head, low muscle tone, language delays and autistic-like social impairments including gaze avoidance.
Question 81
A patient with a prominent jaw, self-hugging behavior, and developmental delay is diagnosed with:
A) Smith-Magenis syndrome
B) Angelman syndrome
C) Cri-du-chat syndrome
D) Cornelia de Lange syndrome
E) Prader-Willi syndrome
Answer: A) Smith-Magenis syndrome
Explanation:Smith-Magenis results from a deletion of 17p11.2. It features broad facial structures (including a prominent jaw), developmental delay, and characteristic self-hugging stereotypies alongside severe self-injury and sleep disturbance.
Question 82
Which feature is characteristic of Prader-Willi syndrome?
A) Hyperphagia and obesity
B) Hypertonia and small hands
C) Seizures and ataxia
D) Immune dysfunction
E) Macroglossia
Answer: A) Hyperphagia and obesity
Explanation:Prader-Willi is caused by loss of paternally expressed genes at 15q11. It presents in infancy with hypotonia and poor feeding, followed by onset of insatiable appetite in childhood leading to severe obesity and related complications.
Question 84
A 10-year-old boy presents with hyperuricemia, self-injurious behavior, and choreoathetosis. Which genetic disorder is most likely?
A) Lesch-Nyhan syndrome
B) Rett syndrome
C) Fragile X syndrome
D) Prader-Willi syndrome
E) Angelman syndrome
Answer: A) Lesch-Nyhan syndrome
Explanation:X-linked HGPRT deficiency causes uric acid accumulation, neurologic signs (choreoathetoid movements), and compulsive self-mutilation beginning in childhood.
Question 85
A 6-year-old girl presents with severe intellectual disability, seizures, and a history of losing previously acquired skills (eg, language). She exhibits stereotypic hand-wringing movements and autonomic dysfunction (eg, irregular breathing). Genetic testing reveals a mutation in the MECP2 gene. Which disorder is MOST likely?
A) Angelman syndrome
B) Rett syndrome
C) Down syndrome
D) Cri-du-chat syndrome
E) Prader-Willi syndrome
Answer: B) Rett syndrome
Explanation:Rett syndrome is due to an X-linked MECP2 mutation, affects girls, and is characterized by normal development until 6–18 months followed by regression of motor and language skills, hand-wringing stereotypies, seizures, and autonomic instability.
Question 86
A 4-year-old girl presents with severe intellectual disability, microcephaly, and a high-pitched cry resembling a cat. On examination, she has hypertelorism and a V-shaped palmar crease. Which genetic disorder is MOST likely?
A) Down syndrome
B) Cri-du-chat syndrome
C) Angelman syndrome
D) Prader-Willi syndrome
E) Williams syndrome
Answer: B) Cri-du-chat syndrome
Explanation:Cri-du-chat arises from a partial deletion of 5p15.2. Infants have a distinctive high-pitched, cat-like cry, pronounced microcephaly, a round face with hypertelorism, and severe learning disability.
Question 87
A 14-year-old girl presents with loss of purposeful hand use, autistic features, and seizures. Genetic testing reveals a mutation in the MECP2 gene. What is the diagnosis?
A) Rett syndrome
B) Angelman syndrome
C) Cri-du-chat syndrome
D) Down syndrome
E) Prader-Willi syndrome
Answer: A) Rett syndrome
Explanation:Rett syndrome is an X-linked dominant disorder caused by mutations in MECP2. After normal early development, girls develop microcephaly, lose purposeful hand movements (replaced by stereotyped hand-wringing), and exhibit autism-like symptoms and seizures.
Question 88
A 6-year-old girl with intellectual disability and facial papules (adenoma sebaceum) is diagnosed with:
A) Neurofibromatosis
B) Tuberous sclerosis
C) Sturge-Weber syndrome
D) Marfan syndrome
E) Ehlers-Danlos syndrome
Answer: B) Tuberous sclerosis
Explanation:Tuberous sclerosis complex (TSC1/TSC2 mutations) causes cortical and retinal hamartomas, seizures, learning disability, and characteristic facial angiofibromas (adenoma sebaceum).
Question 89
A 28-year-old man with intellectual disability, facial angiofibromas, and renal angiomyolipomas is diagnosed with which inherited disorder?
A) Neurofibromatosis type 1
B) Tuberous sclerosis
C) Fragile X syndrome
D) Marfan syndrome
E) Rett syndrome
Answer: B) Tuberous sclerosis
Explanation:Renal angiomyolipomas together with facial angiofibromas and variable learning disability are hallmarks of tuberous sclerosis complex.
Question 90
A 10-year-old boy presents with multiple café-au-lait spots, axillary freckling, and Lisch nodules. Which diagnosis is MOST likely?
A) Neurofibromatosis type 1
B) Tuberous sclerosis
C) Sturge-Weber syndrome
D) von Hippel-Lindau disease
E) Marfan syndrome
Answer: A) Neurofibromatosis type 1
Explanation:NF1 (autosomal dominant, NF1 gene on chromosome 17) is defined by ≥6 café-au-lait macules, axillary/inguinal freckling, and pigmented iris hamartomas (Lisch nodules).
Question 93
A meta-analysis uses funnel plot imputation to address publication bias. If missing studies were excluded, the plot would become asymmetric with smaller negative studies missing. Why was imputation performed?
A) To correct for type II error inflation
B) To balance study arms in a randomized trial
C) To improve representativeness of the evidence base
D) To reduce heterogeneity among included studies
E) To comply with CONSORT reporting standards
Answer: C) To improve representativeness of the evidence base
Explanation: The trim-and-fill procedure imputes the “missing” smaller, negative studies and adds them back into the analysis so that the funnel plot regains symmetry. This corrects underrepresentation of negative results, enhancing the generalizability of the meta-analytic estimate.
Question 94
Which guideline provides standards for reporting systematic reviews and meta-analyses?
A) CONSORT
B) STROBE
C) PRISMA
D) MOOSE
E) STARD
Answer: C) PRISMA
Explanation:PRISMA (Preferred Reporting Items for Systematic Reviews and Meta-Analyses) is the evidence-based minimum checklist that replaced the earlier QUOROM statement in 2009. It outlines the items authors should include to ensure transparent, reproducible reporting of systematic reviews and meta-analyses.
Question 97
A heroin-dependent man stockpiles benzodiazepines and laxatives after his wife demands he quit. This behavior aligns with which stage of change?
A) Pre-contemplation
B) Contemplation
C) Preparation
D) Action
E) Maintenance
Answer: C) Preparation
Explanation: In the preparation stage, individuals acknowledge the need for change and begin to take practical steps—such as gathering supplies—to implement that change imminently. Stockpiling medications for withdrawal is a classic preparation behavior.
Question 98
A 16-year-old smoker who insists “I don’t have a problem” best fits which stage of the Transtheoretical Model?
A) Contemplation
B) Preparation
C) Pre-contemplation
D) Action
E) Relapse
Answer: C) Pre-contemplation
Explanation:Pre-contemplation is characterized by denial or ignorance of the problem and no intention to change within the near future. The statement “I don’t have a problem” typifies this stage.
Question 99
In Prochaska’s Transtheoretical Model, which phase involves actively maintaining behavior change?
A) Pre-contemplation
B) Contemplation
C) Preparation
D) Action
E) Maintenance
Answer: E) Maintenance
Explanation:Maintenance follows action and focuses on sustaining the newly adopted behavior over time and avoiding relapse into old patterns.
Question 100
A 68-year-old woman with Binswanger’s disease would MOST likely exhibit which MRI finding?
A) Hippocampal atrophy
B) Cortical infarcts
C) Diffuse white matter hyperintensities sparing cortex
D) Frontotemporal lobar atrophy
E) Striatal atrophy
Answer: C) Diffuse white matter hyperintensities sparing cortex
Explanation:Binswanger’s disease is a subcortical (small-vessel) vascular dementia characterized by widespread leukoaraiosis—radial, periventricular and deep white matter hyperintensities—while the cortex remains relatively preserved.
Question 101
A 60-year-old woman with Binswanger’s disease presents with gradual memory decline. Which underlying pathology is MOST likely?
A) Beta-amyloid plaques
B) Lacunar infarcts
C) Frontotemporal lobar degeneration
D) Lewy bodies
E) Prion protein accumulation
Answer: B) Lacunar infarcts
Explanation: The hallmark pathology in Binswanger’s disease is multiple small (lacunar) infarcts of penetrating arterioles, producing microvascular ischemic lesions throughout the subcortical white matter.
Question 102
A 72-year-old man with hypertension and diabetes presents with progressive cognitive decline, gait apraxia, and urinary incontinence. MRI shows periventricular leukoaraiosis and lacunar infarcts. What is the most likely diagnosis?
A) Alzheimer’s disease
B) Lewy body dementia
C) Vascular dementia (Binswanger’s type)
D) Frontotemporal dementia
E) Normal pressure hydrocephalus
Answer: C) Vascular dementia (Binswanger’s type)
Explanation: This triad—executive dysfunction, gait disturbance, urinary incontinence—combined with MRI evidence of subcortical white matter hyperintensities and lacunar infarcts, is classic for Binswanger’s (subcortical vascular) dementia.
Question 103
A 65-year-old woman with vascular dementia exhibits apathy and urinary incontinence. Brain MRI shows periventricular white matter hyperintensities. Which subtype is MOST likely?
A) Alzheimer’s disease
B) Dementia with Lewy bodies
C) Frontotemporal dementia
D) Binswanger’s disease
E) Creutzfeldt-Jakob disease
Answer: D) Binswanger’s disease
Explanation:Periventricular leukoaraiosis accompanied by subcortical features (apathy, gait/apraxia, incontinence) defines Binswanger’s subcortical vascular dementia.
Question 104
A 30-year-old man presents with progressive limb ataxia, bilateral nystagmus, and a family history of early myocardial infarction. Genetic testing reveals a trinucleotide repeat expansion on chromosome 9. What is the diagnosis?
A) Huntington’s disease
B) Friedreich’s ataxia
C) Spinocerebellar ataxia type 3
D) Myotonic dystrophy
E) Wilson’s disease
Answer: B) Friedreich’s ataxia
Explanation:Friedreich’s ataxia is an autosomal recessive disorder caused by GAA trinucleotide repeat expansions in the frataxin gene on chromosome 9. It produces cerebellar and posterior column degeneration (ataxia, nystagmus) and hypertrophic cardiomyopathy leading to early cardiac death.
Question 105
A 30-year-old man presents with progressive speech difficulties, gait ataxia, nystagmus, and a family history of a grandfather who died from myocardial infarction at 41. Which genetic disorder is MOST likely?
A) Huntington’s disease
B) Friedreich’s ataxia
C) Spinocerebellar ataxia
D) Marfan syndrome
E) Wilson’s disease
Answer: B) Friedreich’s ataxia
Explanation: The combination of cerebellar ataxia, dysarthria, nystagmus, and a pedigree suggesting cardiomyopathy (early MI in a relative) strongly points to Friedreich’s ataxia, linked to GAA expansions on chromosome 9.
Question 106
A woman with alcohol dependence and chronic back pain on tramadol requires relapse-prevention medication. Which is the safest choice?
A) Naltrexone
B) Acamprosate
C) Diazepam
D) Disulfiram
E) Buprenorphine
Answer: B) Acamprosate
Explanation:
• Acamprosate works on GABA/glutamate balance to reduce alcohol craving and is not an opioid antagonist, so it carries no risk of precipitating opioid withdrawal in patients using tramadol.
• By contrast, naltrexone blocks opioid receptors and would trigger acute withdrawal if given to someone on tramadol.
• Diazepam and buprenorphine are not indicated for alcohol relapse prevention, and disulfiram’s aversive reaction makes it unsuitable when opioid use is ongoing.
Question 107
A patient who has completed opioid detoxification and is now fully abstinent seeks medication to maintain opioid abstinence. Which is most appropriate?
A) Acamprosate
B) Buprenorphine
C) Naltrexone
D) Methadone
E) Lofexidine
Answer: C) Naltrexone
Explanation:
• Naltrexone is a pure opioid antagonist that, once initiated after a sufficient opioid-free interval, blocks the rewarding effects of any opioid taken and so helps sustain abstinence.
• Acamprosate is for alcohol dependence only; buprenorphine and methadone are opioid agonists used during maintenance rather than for abstinence‐focused relapse prevention; lofexidine is for withdrawal symptom relief.
Question 108
A patient with a non-dominant (right) parietal lobe lesion most likely presents with:
A) Finger agnosia
B) Dressing apraxia
C) Expressive dysphasia
D) Anosognosia
E) Gerstmann’s syndrome
Answer: D) Anosognosia
Explanation:
• Anosognosia — unawareness or denial of one’s own neurological deficits — is a hallmark of right (non-dominant) parietal lesions, often manifesting as failure to recognize contralateral weakness.
• Finger agnosia and Gerstmann’s features localize to the dominant parietal lobe; expressive dysphasia to frontal language areas; dressing apraxia can accompany right parietal damage but is less pathognomonic.
Question 109
A 63-year-old man with a right parietal stroke is most likely to exhibit:
A) Expressive dysphasia
B) Dressing apraxia
C) Anosognosia
D) Finger agnosia
E) Visual agnosia
Answer: C) Anosognosia
Explanation:
• Right parietal (non-dominant) damage characteristically leads to unawareness of deficits (anosognosia) and neglect of the left side.
• Finger agnosia and agraphia reflect dominant-lobe involvement; expressive dysphasia to Broca’s area; visual agnosia to occipital/temporal lesions.
Question 110
A 50-year-old woman with frontotemporal dementia (FTD) most commonly shows which speech pattern?
A) Fluent aphasia
B) Apraxia
C) Rigidity
D) Visual hallucinations
E) Bradykinesia
Answer: A) Fluent aphasia
Explanation:
• In the semantic-variant of primary progressive aphasia (a form of FTD), speech remains grammatically fluent but shows loss of word meaning, naming errors, and use of nonspecific substitutes.
• Apraxia and rigidity are motor features of other neurodegenerative syndromes; visual hallucinations point to Lewy body dementia; bradykinesia to parkinsonian disorders.
Question 111
A 55-year-old man develops fluent but nonsensical speech with impaired word comprehension and left-temporal atrophy on MRI. Which FTD subtype is this?
A) Behavioral variant (Pick’s Disease)
B) Primary Progressive Aphasia (Non-fluent/Agrammatic)
C) Primary Progressive Aphasia (Semantic)
D) Logopenic variant
E) Corticobasal syndrome
Answer: C) Primary Progressive Aphasia (Semantic)
Explanation:
• Semantic-variant PPA is marked by fluent output yet profound impairment in word meaning and object naming, with disproportionate anterior temporal atrophy.
• Non-fluent PPA causes effortful, halting speech; logopenic variant features word-finding pauses with preserved grammar; behavioral variant presents early personality change.
Question 112
A 68-year-old man with FTD develops fluent but meaningless speech and poor comprehension. Which diagnosis fits best?
A) Broca’s aphasia
B) Wernicke’s aphasia
C) Primary progressive aphasia
D) Conduction aphasia
E) Global aphasia
Answer: C) Primary progressive aphasia
Explanation:
• Primary progressive aphasia (semantic type) in FTD presents similarly to Wernicke’s—fluent, jargon-filled speech with impaired comprehension—but evolves gradually and is linked to neurodegeneration rather than acute stroke.
Question 113
A 68-year-old man with FTD speaks slowly, with frequent word-finding pauses, but intact single-word comprehension. Which subtype is this?
A) Anomic aphasia
B) Non-fluent (primary motor) dysphasia
C) Wernicke’s aphasia
D) Conduction aphasia
E) Global aphasia
Answer: B) Non-fluent (primary motor) dysphasia
Explanation:
• The non-fluent (agrammatic) variant of primary progressive aphasia is defined by halting, effortful speech and agrammatism, while comprehension of individual words remains relatively spared.
• Anomic aphasia involves isolated word-finding difficulty without apraxia; Wernicke’s spares fluency; conduction aphasia features impaired repetition with fluent speech; global aphasia is rare in gradual dementing illness.
Question 114
A 70-year-old man with vascular dementia and diabetes mellitus develops delirium. His current medications include risperidone and metformin. Which factor is MOST likely contributing to delirium?
A) Hyperglycemia
B) Polypharmacy
C) Dehydration
D) Hypotension
E) Infection
Answer: B) Polypharmacy
Explanation: In older patients, delirium is often precipitated by use of multiple medications. Having more than four drugs increases anticholinergic and psychoactive burden, impairing cognition and precipitating acute confusional states.
Question 115
A 32-year-old man with schizophrenia on clozapine has gained 18 kg in 4 months despite stable psychosis. His BMI is now 32 kg/m². Lifestyle counseling has been offered but not adhered to. What is the next most appropriate intervention?
A) Reduce clozapine dose
B) Initiate metformin
C) Add fluoxetine
D) Switch to aripiprazole
E) Prescribe orlistat
Answer: B) Initiate metformin
Explanation:Metformin is the first-line pharmacotherapy for antipsychotic-induced weight gain. It has consistent evidence for attenuating weight increase without compromising psychiatric stability, whereas dose reduction risks relapse.
Question 116
A patient on haloperidol develops akathisia. The FIRST-line treatment is:
A) Lorazepam
B) Propranolol
C) Benztropine
D) Amantadine
E) Quetiapine
Answer: B) Propranolol
Explanation:Beta-blockers are the preferred initial pharmacologic intervention for neuroleptic-induced akathisia. Propranolol (10 mg tds) alleviates restlessness by blocking peripheral adrenergic receptors, whereas anticholinergics target dystonia, not akathisia.
Question 117
A patient on haloperidol develops restlessness and pacing. The first-line treatment is:
A) Propranolol
B) Tetrabenazine
C) Reduce antipsychotic dose
D) Lorazepam
E) Cyproheptadine
Answer: A) Propranolol
Explanation:Acute akathisia presents as subjective inner restlessness plus motor agitation. Propranolol is more effective and faster-acting than benzodiazepines for this indication, while dose reduction alone may not be feasible immediately.
Question 118
Compared to bulimia nervosa, which of the following is TRUE?
A) Anorexia has a higher lifetime prevalence
B) Bulimia has a younger age of onset
C) Anorexia carries a higher mortality rate
D) Bulimia shows equal male-to-female ratio
E) Binge-eating episodes are shorter in anorexia
Answer: C) Anorexia carries a higher mortality rate
Explanation: Although bulimia is more common, anorexia nervosa has the highest psychiatric mortality—10–12 times greater than controls—driven by medical complications and suicide.
Question 119
Which statement regarding the epidemiology of eating disorders is CORRECT?
A) Anorexia nervosa is more prevalent than bulimia nervosa.
B) Bulimia nervosa typically has an earlier onset than anorexia nervosa.
C) Anorexia nervosa is disproportionately prevalent in lower socioeconomic classes.
D) Bulimia nervosa is approximately twice as common as anorexia nervosa.
E) Cultural globalization has no impact on the incidence of eating disorders.
Answer: D) Bulimia nervosa is approximately twice as common as anorexia nervosa.
Explanation: Epidemiological studies consistently show bulimia’s point prevalence is roughly double that of anorexia, reflecting both higher incidence and underdiagnosis in anorexia.
Question 120
Which statement best describes the prevalence of eating disorders in developed countries?
A) Anorexia nervosa is more common than bulimia nervosa
B) Bulimia nervosa is more common than anorexia nervosa
C) Prevalence is equal across all diagnostic categories
D) Incidence is highest in males aged 18–25 years
E) Prevalence is static across different socioeconomic strata
Answer: B) Bulimia nervosa is more common than anorexia nervosa
Explanation:Bulimia’s higher prevalence reflects both greater symptom recognition and possibly cultural factors; eating-disorder rates also vary by gender and socioeconomic status.
Question 121
A 36-year-old man with depression struggles with motivation. Behavioral activation therapy for him will focus on:
A) Identifying automatic negative thoughts
B) Challenging cognitive distortions
C) Addressing TRAP (Trigger, Response, Avoidance Pattern) cycles
D) Improving sleep hygiene
E) Increasing physical exercise
Answer: C) Addressing TRAP (Trigger, Response, Avoidance Pattern) cycles
Explanation:Behavioral activation breaks the cycle of triggers, maladaptive responses, and avoidance, systematically increasing engagement in reinforcing activities to improve mood.
Question 122
A 30-year-old woman with moderate intellectual disability (ID) presents with aggression toward staff during hygiene routines. A functional behavior assessment (FBA) identifies that aggression occurs when staff insist on bathing her. Which intervention is MOST appropriate?
A) Prescribe risperidone
B) Implement a desensitization program
C) Apply physical restraints during bathing
D) Avoid bathing entirely
E) Administer benzodiazepines PRN
Answer: B) Implement a desensitization program
Explanation:Systematic desensitization gradually exposes the patient to the bathing trigger in a controlled way, reducing avoidance–aggression cycles and teaching coping responses.
Question 123
Key difference between sleep terror and nightmare:
A) Terror occurs in REM sleep
B) Nightmare lacks recall
C) Terror lacks recall
D) Both occur in NREM
E) No physiological arousal
Answer: C) Terror lacks recall
Explanation:Sleep terrors arise in NREM stages and patients have little or no memory of the episode, whereas nightmares from REM sleep are vividly recalled upon awakening.
Question 124
A 78-year-old woman with Lewy body dementia presents with frequent falls and vivid nightmares of being chased. Which sleep disorder is MOST likely contributing to her symptoms?
A) Sleep apnoea
B) REM sleep behavior disorder
C) Narcolepsy
D) Periodic limb movement disorder
E) Insomnia
Answer: B) REM sleep behavior disorder
Explanation: In Lewy body dementia, loss of normal REM atonia leads to dream enactment—violent movements during sleep that cause injury and falls.
Question 125
A 65-year-old man with Parkinson’s disease develops violent leg movements during dreams. His wife reports he shouts and strikes out. Which condition is MOST likely?
A) Obstructive sleep apnoea
B) REM sleep behavior disorder
C) Nocturnal seizures
D) Night terrors
E) Delirium
Answer: B) REM sleep behavior disorder
Explanation:Dream enactment behavior with vocalizations and motor movements is characteristic of REM sleep behavior disorder, strongly linked to synucleinopathies.
Question 126
A meta-analysis uses funnel plot imputation to address publication bias. If missing studies were excluded, the plot would become asymmetric with smaller negative studies missing. Why was imputation performed?
A) To correct for type II error inflation
B) To balance study arms in a randomized trial
C) To improve representativeness of the evidence base
D) To reduce heterogeneity among included studies
E) To comply with CONSORT reporting standards
Answer: C) To improve representativeness of the evidence base
Explanation:Imputing missing negative studies reduces bias from preferential publication of positive results, yielding a more accurate overall estimate of effect.
Question 127
A researcher plots study results from multiple RCTs to assess heterogeneity and publication bias. Which graph is MOST suitable?
A) Forest plot
B) Funnel plot
C) Box plot
D) Scatter plot
E) Kaplan-Meier plot
Answer: B) Funnel plot
Explanation:Funnel plots display effect size against study precision; symmetry suggests low publication bias, while asymmetry indicates missing small negative studies.
MRCPsych Paper B - 21 May 2025
MRCPsych Paper B - 21 May 2025
Question 1
Dundrum Crisis Scale primarily assesses:
A) Suicidal intent
B) Risk of violence
C) Functional impairment
D) Insight
E) Medication adherence
Answer: A. Suicidal intent
Explanation:
This scale helps evaluate the immediate danger of self-harm in patients in a psychiatric setting when assessing Suicidal intent.
Question 10
In evidence-based hierarchy, which study design is ranked HIGHEST?
A) Case-control
B) Cohort
C) Cross-sectional
D) Randomized controlled trial (RCT)
E) Systematic review
Answer: E. Systematic review
Explanation:
Systematic reviews synthesize results from multiple RCTs or observational studies, applying rigorous methods to minimize bias. Because they integrate all high-quality evidence on a question, they sit at the top of the evidence pyramid. Individual RCTs come next, then cohort and case-control studies, with cross-sectional near the bottom.
Question 11
A meta-analysis assumes fixed effects when:
A) Study heterogeneity is high
B) Effect sizes vary significantly
C) Population and interventions are homogeneous
D) Publication bias is present
E) Sample sizes are small
Answer: C. Population and interventions are homogeneous
Explanation:
A fixed-effect model treats all studies as estimating the same true effect size under identical conditions. That assumption only makes sense when subjects, interventions, and outcomes are sufficiently similar across studies—i.e., minimal heterogeneity. Thus, it's when Population and interventions are homogeneous.
Question 12
Which measure of central tendency is MOST appropriate for skewed data (e.g., income distribution)?
A) Mean
B) Median
C) Mode
D) Range
E) Standard deviation
Answer: B. Median
Explanation:
The Median marks the 50th percentile and isn’t influenced by extreme values. In skewed distributions—where a few very large or small observations would drag the mean away from the center—the median remains the best single-value summary of the “typical” case.
Question 13
Negative skew in a dataset is indicated by:
A) Mean > Mode
B) Mean < Mode
C) Median = Mean
D) Long right tail
E) Symmetrical distribution
Answer: B. Mean < Mode
Explanation:
Negative skew means the left tail extends farther than the right. That pulls the arithmetic mean below the mode (hence Mean < Mode), since extreme low values drag down the average more than the most common value.
Question 14
A physician decides to work at a rural clinic instead of a city hospital. This decision BEST illustrates:
A) Sunk cost fallacy
B) Opportunity cost
C) Marginal utility
D) Diminishing returns
E) Cost-benefit analysis
Answer: B. Opportunity cost
Explanation:
Opportunity cost is the value of the next-best alternative you give up. By choosing the rural clinic, the physician sacrifices whatever salary, caseload, or career development they could have had in the city. Recognizing that forgone benefit is exactly the concept of Opportunity cost.
Question 15
A researcher validates a new depression screening tool against the PHQ-9. Which type of validity are they assessing?
A) Predictive validity
B) Concurrent validity
C) Content validity
D) Construct validity
E) Criterion validity
Answer: B. Concurrent validity
Explanation:
Concurrent validity checks how well a new measure correlates with an established “gold standard” when both are administered at the same time. Here, comparing the new tool’s scores directly against PHQ-9 results in the same sample assesses Concurrent validity.
Question 2
A clinician adjusts the CAGE cut-off score from 2 to 3. What is the most likely effect on its diagnostic metrics?
A) Increased sensitivity, decreased specificity
B) Decreased sensitivity, increased specificity
C) Increased positive predictive value, unchanged negative predictive value
Answer: B. Decreased sensitivity, increased specificity
Explanation:
The CAGE questionnaire gives one point for each “yes” answer across its four items. By raising the threshold from two affirmative responses to three, you make it harder to screen positive. As a result, fewer true cases of alcohol misuse will be flagged (sensitivity falls), but you also reduce the number of false positives (specificity rises). In other words, you miss more people who do have a problem but you’re less likely to wrongly label someone as disordered. This results in Decreased sensitivity, increased specificity.
Question 3
A researcher conducts a study to evaluate a new depression screening tool. They report a P-value of 0.04. Which statement accurately interprets this result?
A) There is a 4% probability the null hypothesis is true.
B) The tool’s diagnostic accuracy is 96% reliable.
C) There is a 4% chance of falsely rejecting the null hypothesis.
D) The effect size is 0.04.
E) The study has 96% power.
Answer: C. There is a 4% chance of falsely rejecting the null hypothesis.
Explanation:
A P-value of 0.04 means that if in reality there were no effect (the null hypothesis is true), you would obtain data at least as extreme as observed in only 4 out of 100 repeated experiments. It does not tell you the probability that the null hypothesis itself is true, nor does it speak to accuracy, effect size, or power directly. Therefore, there is a 4% chance of falsely rejecting the null hypothesis (Type I error rate if the null is true).
Question 4
A study finds no difference in depression scores between two therapies but reports significant improvements in sleep as a secondary outcome. This MOST likely indicates:
A) Reporting bias
B) Confounding bias
C) Selection bias
D) Observer bias
E) Random error
Answer: A. Reporting bias
Explanation:
When researchers highlight favorable secondary outcomes (like sleep) despite a negative primary endpoint (depression scores), they risk giving a skewed picture of benefit. This selective emphasis—reporting only the “good news”—is classic Reporting bias. Other biases (confounding, selection, observer) involve design or measurement flaws, not selective presentation of results.
Question 5
In optimizing the CAGE questionnaire’s likelihood ratio for detecting harmful alcohol use, which cutoff score provides the highest positive likelihood ratio (LR+)?
A) 0 (any affirmative response)
B) 1
C) 2
D) 3
E) 4
Answer: C. 2
Explanation:
The positive likelihood ratio (LR+) equals sensitivity ÷ (1 – specificity). At a cutoff of two “yes” answers, the CAGE typically yields about 72% sensitivity and 75% specificity, giving an LR+ around 2.9. Lower cutoffs boost sensitivity but harm specificity, reducing LR+. Higher cutoffs improve specificity further but sacrifice too much sensitivity, again lowering LR+. Thus, a cutoff of 2 strikes the best overall balance for ruling in harmful use.
Question 6
Which statistical graph is used in meta-analyses to visually assess heterogeneity across studies?
A) Forest plot
B) Galbraith plot
C) Funnel plot
D) Scatterplot
E) Box plot
Answer: B. Galbraith plot
Explanation:
A Galbraith plot (also called a radial plot) graphs each study’s standardized effect size against its precision (the inverse of its standard error). Outlier studies will stand apart from the main cluster, flagging heterogeneity. While Forest plots summarize individual results and Funnel plots detect publication bias, only the Galbraith plot directly highlights between-study variability.
Question 7
What is the PRIMARY purpose of a Galbraith plot in systematic reviews?
A) Compare treatment effects
B) Assess publication bias
C) Evaluate heterogeneity
D) Visualize publication dates
E) Calculate odds ratios
Answer: C. Evaluate heterogeneity
Explanation:
Galbraith plots are specifically designed to flag heterogeneity by showing which studies deviate most from the average effect size once you adjust for precision. They help reviewers decide whether a fixed-effect or random-effects model is more appropriate, thus their primary purpose is to Evaluate heterogeneity.
Question 8
A patient answers “No” to 3 CAGE questions. What is the BEST interpretation?
A) High specificity for alcohol dependence
B) Moderate sensitivity for hazardous drinking
C) Low probability of alcohol use disorder
D) Confirmatory evidence of sobriety
E) Indication for liver function testing
Answer: C. Low probability of alcohol use disorder
Explanation:
A total score of zero on the CAGE (three “No” answers plus one remaining, assuming the question implies only one "Yes" or zero "Yes" answers if 3 are "No") yields high specificity—meaning few healthy people will test positive—and so a negative result effectively rules out disorder. It doesn’t absolutely confirm sobriety, but it does make alcohol use disorder unlikely, indicating a Low probability of alcohol use disorder.
Question 17
A study compares pain levels in three groups: medication A, B, and C. Which statistical test is MOST appropriate?
A) Unpaired t-test
B) Mann-Whitney U test
C) Kruskal-Wallis test
D) ANOVA
E) Chi-square test
Answer: C. Kruskal-Wallis test
Explanation:
The Kruskal-Wallis test is the non-parametric equivalent of one-way ANOVA and is used to compare three or more independent groups when the outcome (e.g., pain scores) may not follow a normal distribution. It ranks all observations and tests whether the distributions of ranks differ across groups, making it ideal for skewed or ordinal data without assuming equal variances or normality.
Question 18
A researcher compares baseline characteristics between treatment and control groups. Which test adjusts for confounding variables like age and sex?
A) Unpaired t-test
B) ANCOVA
C) Chi-square test
D) Fisher’s exact test
E) Wilcoxon signed-rank test
Answer: B. ANCOVA
Explanation:
Analysis of covariance (ANCOVA) combines ANOVA and regression: it compares means of a continuous outcome across groups while statistically controlling for one or more continuous or categorical covariates (e.g., age, sex). By including confounders in the model, ANCOVA isolates the effect of the main group factor on the outcome, reducing bias from imbalanced baseline characteristics.
Question 19
A psychiatrist compares pre- and post-intervention depression scores in two independent groups. Which test is BEST?
A) Paired t-test
B) Unpaired t-test
C) Mann-Whitney U test
D) Wilcoxon signed-rank test
E) Repeated-measures ANOVA
Answer: B. Unpaired t-test
Explanation:
When comparing the means of a continuous outcome between two independent groups at a single time point (e.g., post-intervention scores, or change scores if pre-subtracted), the two-sample (Unpaired t-test) is appropriate, provided the data are approximately normally distributed and variances are similar. It tests whether the difference in group means is statistically significant.
Question 20
A 2×2 contingency table shows a rare outcome (<5% frequency). Which test is MOST appropriate?
A) Chi-square test
B) Fisher’s exact test
C) McNemar’s test
D) Cochran’s Q test
E) Mantel-Haenszel test
Answer: B. Fisher’s exact test
Explanation:
In a 2×2 contingency table, if the expected frequency in any cell is less than 5 (common when outcomes are rare), the chi-square approximation becomes unreliable. Fisher’s exact test computes the exact probability of observing the data under the null hypothesis and is the standard method for small sample or low-frequency tables.
Question 21
A table reports adjusted odds ratio (OR = 1.5) and unadjusted OR = 2.0. Why is the adjusted OR lower?
A) Confounding by age
B) Smaller sample size
C) Type I error
D) Measurement bias
E) Random variation
Answer: A. Confounding by age
Explanation:
An unadjusted odds ratio may overestimate an association if a confounding variable (e.g., age) is unevenly distributed between groups and is associated with both the exposure and outcome. By including age as a covariate in a multivariable logistic regression model, the adjusted OR accounts for its effect, often attenuating the estimate when age was positively associated with both exposure and outcome. This is described as Confounding by age.
Question 27
Which study design best examines the association between lithium levels and depression severity?
A) Case-control
B) Cohort
C) Ecological
D) Cross-sectional
E) Meta-analysis
Answer: B. Cohort
Explanation:
A prospective cohort study enrolls participants based on exposure status (e.g., different lithium levels) and follows them over time to measure subsequent outcomes (depression severity). This design establishes temporal sequence, reduces recall bias, and allows direct measurement of incidence and risk related to the exposure. Thus, a Cohort study is most appropriate.
Question 28
Which study design is initially most appropriate to evaluate side effects of a new drug in 3 exposed individuals?
A) Case series
B) Case-control
C) N-of-1 trial
D) Cohort
E) Randomized controlled trial
Answer: A. Case series
Explanation:
A Case series describes and analyzes the clinical course of a small group of patients who share a specific exposure (here, 3 individuals on the new drug). It is a descriptive study useful for early identification of potential side effects before larger, more formal studies are undertaken.
Question 32
Two radiologists interpreting MRI results are assessing:
A. Concurrent validity
B. Content validity
C. Inter-rater reliability
D. Predictive validity
E. Internal consistency
Answer: C. Inter-rater reliability
Explanation (quick-revision points):
• Reliability deals with consistency; inter-rater reliability is consistency between observers rating the same cases.
• In psychometrics it is commonly quantified with κ (kappa) for categorical ratings or the intraclass correlation coefficient for continuous data.
A suicide risk tool’s scores correlate with future suicide attempts. This demonstrates:
A. Predictive validity
B. Concurrent validity
C. Content validity
D. Construct validity
E. Face validity
Answer: A. Predictive validity
Explanation:
• Predictive validity asks: does today’s score forecast a later, clinically meaningful outcome?
• Criterion validity has two flavours – concurrent (same time-point) and predictive (future event).
• High scores that accurately anticipate later self-harm indicate sound predictive validity.
Question 34
An ROC curve with an AUC of 0.86 and sensitivity of 100 % indicates:
A) Poor specificity
B) Optimal screening accuracy
C) High false-positive rate
D) Inadequate discriminative ability
E) Superior positive predictive value
Answer: B. Optimal screening accuracy
Explanation:
• AUC > 0.8 = excellent overall discrimination; a value of 0.86 is well inside this zone.
• Sensitivity 100 % means no diseased cases are missed. Where the curve hugs the top-left corner the test achieves its best balance.
• Specificity can still vary at different cut-offs, but the combination of very high sensitivity and high AUC reflects near-ideal screening performance, indicating Optimal screening accuracy.
Question 35
A loneliness measure correlates with depression (r = 0.38). What does this indicate?
A) Depression causes loneliness
B) A moderate positive association exists
C) Loneliness predicts depression onset
D) Shared measurement bias
E) No clinically meaningful relationship
Answer: B. A moderate positive association exists
Explanation:
Pearson’s correlation coefficient (r) ranges from –1 to +1, with values nearer ±1 indicating stronger linear relationships. An r of 0.38 falls in the moderate range, signifying that higher loneliness scores tend to occur alongside higher depression scores, but causality cannot be inferred from correlation alone.
Question 36
A study investigates hours of sleep vs. cognitive performance in older adults. The correlation coefficient is reported as –0.85. What does this indicate?
A) Strong positive linear relationship
B) Weak negative linear relationship
C) No relationship
D) Strong negative linear relationship
E) Perfect inverse relationship
Answer: D. Strong negative linear relationship
Explanation:
Values of r near –1 indicate strong inverse linear associations. An r of –0.85 means that as sleep hours decrease, cognitive performance tends to decrease in a strongly linear fashion. Pearson’s r quantifies both direction (sign) and strength (magnitude) of linear relationships.
Question 37
A depressed inpatient scores highly on both Scale A (100-item) and Scale B (12-item) with Spearman’s ρ = 0.8 (p < 0.01). Which conclusion is MOST valid?
A) Scale B is a reliable replacement for Scale A
B) Both scales show excellent inter-rater reliability
C) A and B are highly positively correlated
D) Scale A is more valid than Scale B
E) No conclusion can be drawn about validity
Answer: C. A and B are highly positively correlated
Explanation:
Spearman’s rho measures the strength of a monotonic association between two ranked variables without assuming normality. A ρ of 0.8 indicates a strong positive correlation, meaning both scales track the same construct closely—but it says nothing about validity, reliability, or replacement of one by the other.
Question 38
Two depression screening tools show Spearman’s ρ = 0.81. What does this imply?
A) High inter-rater reliability
B) Strong positive correlation
C) Good predictive validity
D) High sensitivity
E) Low specificity
Answer: B. Strong positive correlation
Explanation:
A Spearman’s rho of 0.81 denotes a strong positive monotonic relationship between the two instruments, indicating they yield very similar rankings of depressive severity across subjects.
Question 39
An r value of –1 indicates:
A) Perfect positive correlation
B) Perfect negative correlation
C) Moderate positive correlation
D) No correlation
E) Weak negative correlation
Answer: B. Perfect negative correlation
Explanation:
A Pearson r of –1 signifies a perfect inverse linear relationship: every increase in X corresponds to a proportional decrease in Y, and all data points lie exactly on a downward-sloping line.
Question 40
A researcher finds a correlation coefficient of –0.9 between alcohol consumption and liver enzyme levels. What does this indicate?
A) Strong inverse relationship
B) Weak direct relationship
C) No relationship
D) Moderate inverse relationship
E) Perfect direct relationship
Answer: A. Strong inverse relationship
Explanation:
An r of –0.9 is very close to –1, reflecting a very strong negative linear association. Here, higher alcohol intake is strongly linked with higher liver enzyme levels in an inverse coding scheme (e.g., if enzymes were scored inversely).
Question 41
In a scatter plot of age vs. cognitive scores, the trend line has an R value of 0.7. What does this indicate?
A) Weak positive correlation
B) Strong positive correlation
C) Weak negative correlation
D) Strong negative correlation
E) No correlation
Answer: B. Strong positive correlation
Explanation:
An R (Pearson’s correlation) of 0.7 indicates a strong positive linear association in the chosen coding: as one variable increases, so does the other. The closer |R| is to 1, the stronger the linear link.
Question 42
A Kendall’s tau-b coefficient of 0.45 is reported between pain severity (ordinal) and sleep quality (ordinal). What does this indicate?
A) No association
B) Moderate positive association
C) Pain explains 45% of sleep variance
D) The association is coincidental
E) A parametric test would yield a stronger correlation
Answer: B. Moderate positive association
Explanation:
Kendall’s tau-b ranges from –1 to +1, with values around 0.4–0.6 indicating moderate associations between ordinal variables. A τ of 0.45 therefore reflects a clear, moderate positive relationship.
Question 43
A researcher uses Kendall’s coefficient for depression severity (Likert) vs. quality of life (ordinal). What is the PRIMARY advantage of this method?
A) Assumes normal distribution
B) Measures non-linear associations
C) Handles tied ranks without bias
D) Compares group means
E) Requires parametric assumptions
Answer: C. Handles tied ranks without bias
Explanation:
Unlike Spearman’s rho—which assumes equal spacing between ranks—Kendall’s tau-b does not require equidistant ranking and properly accounts for tied observations in ordinal data.
Question 44
Which statistical test is MOST appropriate for assessing the strength and direction of a linear relationship between two continuous variables?
A) Spearman’s rank correlation
B) Pearson’s correlation
C) Chi-square test
D) Student’s t-test
E) ANOVA
Answer: B. Pearson’s correlation
Explanation:
Pearson’s r quantifies the linear association between two interval/ratio-scale variables under normality assumptions, providing both direction and magnitude of the relationship.
Question 45
Which statistical method is Kendall’s coefficient used for?
A) Parametric correlation
B) Non-parametric correlation for ordinal data
C) Paired t-test
D) ANOVA
E) Regression analysis
Answer: B. Non-parametric correlation for ordinal data
Explanation:
Kendall’s tau is a non-parametric measure of association suited to ordinal variables or small sample sizes, offering a robust alternative when rank distributions aren’t equidistant.
Question 46
A study on childhood sexual abuse vs. adult depression reports r = 0.35 (p = 0.04). Which interpretation is MOST accurate?
A) 35% chance abuse causes depression
B) Statistically significant association, but causality unclear
C) Abuse explains 35% of depression cases
D) Result due to chance
E) Larger sample would eliminate the association
Answer: B. Statistically significant association, but causality unclear
Explanation:
A p-value <0.05 confirms a real association unlikely due to chance, but correlation does not establish cause–effect and may reflect confounders. Correlation quantifies strength/direction, not causation.
Question 47
A researcher compares baseline HAMD scores across three timepoints. Which statistical method is MOST suitable?
A) Repeated-measures ANOVA
B) Mann-Whitney U test
C) Pearson correlation
D) Chi-square test
E) Linear regression
Answer: A. Repeated-measures ANOVA
Explanation:
For within-subject comparisons at ≥3 timepoints on a continuous measure, repeated-measures ANOVA accounts for the correlation between repeated observations and tests for overall change over time.
Question 48
A longitudinal study assesses behavioral problems at ages 3, 5, and 7 linked to bedtime schedules, reporting regression coefficients (β = –0.8, p < 0.01; β = –0.6, p = 0.03; β = –0.4, p = 0.12). Which analysis is MOST appropriate?
A) Logistic regression
B) Linear regression
C) Cox proportional hazards
D) Chi-square test
E) ANOVA
Answer: B. Linear regression
Explanation:
Continuous outcomes modeled against continuous predictors at multiple timepoints are examined via linear regression, which provides β coefficients and p-values for each predictor–outcome relationship.
Question 49
A study evaluates blood pressure (BP) and cognitive decline in older adults, adjusting for age. Which statistical method is MOST appropriate?
A) Linear regression with BP as the dependent variable
B) Logistic regression with cognitive decline as binary
C) Multiple linear regression including age as a covariate
D) Spearman’s correlation
E) ANOVA
Answer: C. Multiple linear regression including age as a covariate
Explanation:
Multiple linear regression allows estimation of the independent effect of BP on cognitive scores while statistically controlling for confounders like age by including them in the model.
Question 50
A study analyzes time-to-event data for patients with heart failure. Which statistical method is MOST appropriate?
A) Cox proportional hazards model
B) Logistic regression
C) Linear regression
D) ANOVA
E) Chi-square test
Answer: A. Cox proportional hazards model
Explanation:
Cox regression is the standard survival-analysis technique for modeling the hazard (instantaneous risk) of an event over time while allowing adjustment for covariates.
Question 51
Why is logistic regression used in a study predicting diabetes diagnosis (binary outcome)?
A) To model continuous outcomes
B) To handle multicollinearity
C) To assess proportional hazards
D) To predict binary outcomes
E) To adjust for confounding
Answer: D. To predict binary outcomes
Explanation:
Logistic regression estimates the probability of a dichotomous event (e.g., diabetic vs. non-diabetic), yielding odds ratios for predictors without requiring normally distributed residuals.
Question 52
Two radiologists interpret MRI results independently. Which property are they assessing?
A) Concurrent validity
B) Content validity
C) Inter-rater reliability
D) Predictive validity
E) Internal consistency
Answer: C. Inter-rater reliability
Explanation:
Inter-rater reliability quantifies agreement between different observers rating the same subjects using the same instrument, ensuring consistency across raters.
Question 53
A suicide-risk tool’s scores correlate with future suicide attempts. This demonstrates:
A) Predictive validity
B) Concurrent validity
C) Content validity
D) Construct validity
E) Face validity
Answer: A. Predictive validity
Explanation:
Predictive validity is established when a measure accurately forecasts later outcomes—in this case, tool scores predicting subsequent suicide attempts.
Question 54
A researcher validates a new depression screening tool against the PHQ-9 (administered simultaneously). Which validity is being assessed?
A) Predictive validity
B) Concurrent validity
C) Content validity
D) Construct validity
E) Criterion validity
Answer: B. Concurrent validity
Explanation:
Concurrent validity is a subtype of criterion validity, determined by correlating a new measure with an established “gold standard” at the same point in time (here, the PHQ-9).
Question 55
A new anxiety scale is tested alongside clinician diagnoses. Which validity ensures it covers all aspects of anxiety?
A) Concurrent validity
B) Predictive validity
C) Content validity
D) Discriminant validity
E) Test-retest validity
Answer: C. Content validity
Explanation:
Content validity assesses whether a measure includes all the important domains of its intended construct (anxiety) as defined by expert consensus.
Question 56
A clinician administers a new bipolar-disorder screener and the SCID concurrently. Which validity is assessed?
A) Predictive validity
B) Concurrent validity
C) Construct validity
D) Content validity
E) Divergent validity
Answer: B. Concurrent validity
Explanation:
By comparing the new instrument against the Structured Clinical Interview for DSM (SCID) at the same time, one evaluates concurrent validity—a form of criterion validity.
Question 57
An ADHD screener shows high correlation with autism-spectrum measures. Which validity is MOST compromised?
A) Convergent validity
B) Divergent validity
C) Criterion validity
D) Internal consistency
E) Face validity
Answer: B. Divergent validity
Explanation:
Divergent (discriminant) validity is violated when a tool correlates strongly with measures of unrelated constructs—in this case, autism traits—indicating poor discrimination.
Question 58
If a new depression scale shows perfect correlation with an existing validated tool, which validity is demonstrated?
A) Face validity
B) Predictive validity
C) Concurrent validity
D) Construct validity
E) Ecological validity
Answer: C. Concurrent validity
Explanation:
Concurrent validity refers to the degree to which a new measure correlates with an established “gold-standard” instrument when both are administered at the same time.
Question 59
A researcher validates a new depression screening tool by comparing its scores to a clinician’s diagnosis made simultaneously. This evaluates which type of validity?
A) Predictive
B) Concurrent
C) Content
D) Construct
E) Criterion-related
Answer: B. Concurrent validity
Explanation:
By measuring agreement with a gold-standard diagnosis at the same time point, the study is assessing concurrent validity rather than future prediction or content coverage.
Question 60
A researcher validates a new depression screening tool by administering it alongside an established, validated questionnaire during the same assessment period. This method primarily evaluates:
A) Predictive validity
B) Concurrent validity
C) Content validity
D) Construct validity
E) Incremental validity
Answer: B. Concurrent validity
Explanation:
Direct comparison with an existing validated scale at the same time establishes concurrent validity. Predictive validity would require following participants forward to see who develops an outcome later.
Question 61
In a longitudinal study, scores on a cognitive test administered at age 10 are correlated with academic achievement at age 20. This design primarily examines:
A) Content validity
B) Predictive validity
C) Concurrent validity
D) Divergent validity
E) Internal consistency
Answer: B. Predictive validity
Explanation:
Predictive validity assesses how well an instrument forecasts future outcomes. Here, early cognitive scores are used to predict later academic performance.
Question 62
What is the BEST screening tool for alcohol use disorders in primary care?
A) GGT
B) CDT
C) MCV
D) AUDIT
E) Breathalyser
Answer: D. AUDIT
Explanation:
The World Health Organization–recommended Alcohol Use Disorders Identification Test (AUDIT) is a 10-item questionnaire validated in primary-care settings, demonstrating superior sensitivity and specificity compared to brief biochemical markers or single‐question screens.
Question 63
Which therapy routinely incorporates telephone contact as a core component?
A) Dialectical behaviour therapy
B) Cognitive behavioural therapy
C) Psychoanalytic psychotherapy
D) Interpersonal psychotherapy
E) Cognitive analytic therapy
Answer: A. Dialectical behaviour therapy
Explanation:
DBT includes scheduled telephone coaching between sessions to help patients apply emotion‐regulation and distress‐tolerance skills in real‐life crises.
Question 64
Past family therapy approaches were most influenced by which therapeutic tradition?
A) Cognitive-behavioural therapy
B) Psychodynamic psychotherapy
C) Humanistic therapy
D) Systemic therapy
E) Solution-focused therapy
Answer: B. Psychodynamic psychotherapy
Explanation:
Early models of family therapy (e.g., Bowenian) were rooted in psychodynamic principles, emphasizing unconscious conflicts and transgenerational dynamics.
Question 65
Sequential diagrammatic formulation (SD), a tool in CAT, involves which three stages?
A) Reformulation, recognition, revision
B) Cognitive restructuring, exposure, response prevention
CAT is delivered in three phases—initial problem‐formulation (reformulation), patient insight and identification of reciprocal‐role patterns (recognition), and development of new behavioural strategies (revision).
Question 66
The “3 R’s” in Cognitive Analytic Therapy are:
A) Reflection, Reconnection, Resolution
B) Reformulation, Recognition, Revision
C) Relabeling, Restructuring, Reinforcement
D) Regression, Reintegration, Realignment
E) Reassurance, Responsibility, Reconciliation
Answer: B. Reformulation, Recognition, Revision
Explanation:
These steps guide the CAT process: first reformulate the problem in diagram form, then help the patient recognize maladaptive scripts, and finally revise them into healthier patterns.
Question 67
Which of the following is a core “R” in CAT?
A) Reformulation
B) Role Reversal
C) Recognition
D) Revision
E) Recall
Answer: D. Revision
Explanation:
Revision is the active phase where the patient experiments with and adopts new, adaptive ways of relating, replacing old problematic scripts.
Question 68
Which intervention is MOST effective for emotional dysregulation in borderline personality disorder?
A) Flooding
B) Systematic desensitization
C) Dialectical behavior therapy (DBT)
D) Cognitive‐behavioural therapy (CBT)
E) Risperidone
Answer: C. Dialectical behavior therapy (DBT)
Explanation:
DBT was specifically developed to teach emotion‐regulation, distress‐tolerance, interpersonal‐effectiveness, and mindfulness skills, and has the strongest evidence for reducing self‐harm and emotional dysregulation in BPD.
Question 69
A core technique in Motivational Interviewing is:
A) Rolling with resistance
B) Amplifying cognitive dissonance
C) Contingency management
D) Developing discrepancy
E) Avoidance restructuring
Answer: D. Developing discrepancy
Explanation:
MI works by helping patients perceive the gap between their current behaviors and broader goals/values, thereby enhancing intrinsic motivation to change.
Question 70
A motivational interviewing approach to substance misuse would LEAST likely involve:
A) Exploring ambivalence
B) Rolling with resistance
C) Providing tangible rewards for abstinence
D) Eliciting intrinsic motivation
E) Collaborative goal-setting
Answer: C. Providing tangible rewards for abstinence
Explanation:
MI is a client‐centred, directive style focused on resolving ambivalence; it does not include external contingency‐management techniques like tangible rewards.
Question 71
A psychotherapy model focusing on past family experiences is:
A) Couples behaviour therapy
B) Psychodynamic psychotherapy
C) Cognitive-behavioural therapy
D) Family systems therapy
E) Solution-focused therapy
Answer: B. Psychodynamic psychotherapy
Explanation:
Psychodynamic psychotherapy explores early family relationships and unconscious conflicts that shape current emotions and behaviors.
Question 72
Regarding brief IPT, which statement is CORRECT?
A) Sessions last 90 minutes weekly for 6 weeks
B) Focuses on past traumas
C) Requires 1-hour sessions for 12–16 weeks
D) Avoids transference work
E) Uses cognitive restructuring
Answer: C. Requires 1-hour sessions for 12–16 weeks
Explanation:
Standard IPT for depression is delivered in 12–16 weekly, 1-hour sessions, targeting current interpersonal issues rather than historical trauma.
Question 73
IPT for depression primarily focuses on addressing:
A) Cognitive distortions
B) Grief and interpersonal role disputes
C) Behavioral activation
D) Sleep hygiene
E) Trauma processing
Answer: B. Grief and interpersonal role disputes
Explanation:
IPT works on four problem areas—bereavement, role disputes, role transitions, and interpersonal deficits—with grief and disputes being central for depressive symptoms.
Question 74
IPT for grief focuses on:
A) Cognitive restructuring of negative thoughts
B) Enhancing interpersonal relationships and social support
C) Behavioral activation through exposure
D) Mindfulness and acceptance
E) Psychodynamic exploration of past trauma
Answer: B. Enhancing interpersonal relationships and social support
Explanation:
Grief‐focused IPT strengthens social networks and communication to help patients process loss and regain social functioning.
Question 75
A patient displays intense anger toward their therapist, accusing them of betrayal. This MOST likely reflects:
A) Countertransference
B) Transference
C) Projection
D) Acting out
E) Resistance
Answer: B. Transference
Explanation:
Transference occurs when a patient unconsciously redirects feelings about significant others onto the therapist.
Question 76
A therapist feels intense anger toward a patient discussing abusive relationships. This emotional reaction is best termed:
A) Transference
B) Countertransference
C) Projection
D) Displacement
E) Reaction formation
Answer: B. Countertransference
Explanation:
Countertransference is the therapist’s own unconscious emotional response to the patient’s material.
Question 77
Countertransference refers to:
A) The patient’s feelings toward the therapist
B) The therapist’s unconscious emotional response to the patient
C) Transference of past relationships onto the therapist
D) Resistance to treatment
E) A defense mechanism
Answer: B. The therapist’s unconscious emotional response to the patient
Explanation:
By definition, countertransference comprises all of the therapist’s emotional reactions—both conscious and unconscious—to the patient’s transference and material.
Question 78
A 28-year-old woman with congenital long QT syndrome presents with syncope. Her ECG reveals a significantly prolonged QTc interval. Which electrolyte disturbance is most likely contributing to this finding?
A. Hypocalcaemia
B. Hyperkalaemia
C. Hypomagnesaemia
D. Hypophosphataemia
E. Hypercalcaemia
Answer: A. Hypocalcaemia
Explanation:
Hypocalcaemia prolongs phase 2 of the cardiac action potential, lengthening the QT interval. It is one of the key metabolic risk factors for QTc prolongation, alongside hypokalaemia and hypomagnesaemia.
Question 79
A 70-year-old woman with Alzheimer's dementia taking donepezil faints. Her ECG shows regular sinus rhythm with a heart rate of 45 bpm. What is the MOST likely cause?
A. Sinus tachycardia
B. Sinus bradycardia
C. Atrial fibrillation
D. Ventricular tachycardia
E. Complete heart block
Answer: B. Sinus bradycardia
Explanation:
Donepezil, an acetylcholinesterase inhibitor, enhances vagal tone and can lead to sinus bradycardia, sometimes resulting in syncope in older patients.
Question 80
An 85-year-old man on donepezil develops dizziness and a sinus bradycardia (HR 45 bpm). Which ECG finding is MOST consistent with his medication?
A. QT prolongation
B. Sinus bradycardia
C. Atrial fibrillation
D. ST elevation
E. Right axis deviation
Answer: B. Sinus bradycardia
Explanation:
Bradycardia is a well-recognized vagotonic effect of cholinesterase inhibitors like donepezil, reflecting increased parasympathetic activity on the sinoatrial node.
Question 81
An 80-year-old on donepezil 10 mg reports nausea and dizziness. The next step is:
A. Add an antiemetic
B. Switch to rivastigmine
C. Reduce dose to 5 mg
D. Discontinue memantine
E. Initiate beta-blockers
Answer: C. Reduce dose to 5 mg
Explanation:
Gastrointestinal and vestibular side effects of donepezil are dose-related and often transient. The first-line approach is to lower the dose before switching agents or adding other medications.
Question 82
A patient on lithium presents with polyuria, tremor, and myoclonus. Serum lithium level is 1.5 mmol/L. Which toxicity sign is MOST concerning?
A) Polyuria (>3 L/day)
B) Fine tremor
C) Myoclonic jerks
D) Abdominal pain
E) Confusion
Answer: C. Myoclonic jerks
Explanation:
At levels ≥1.5 mmol/L, lithium toxicity begins to manifest with gastrointestinal upset and central nervous system effects such as coarse tremor, ataxia, drowsiness, confusion, and muscle twitching (myoclonus). Myoclonic jerks signal severe neurotoxicity and warrant immediate intervention.
Question 83
A patient with lithium toxicity presents with coarse tremors and nystagmus. Which of the following is LEAST likely to be a feature?
A) Ataxia
B) Dysarthria
C) Hyporeflexia
D) Confusion
E) Seizures
Answer: C. Hyporeflexia
Explanation:
Lithium neurotoxicity produces hyperactive neurological signs—coarse tremor, ataxia, dysarthria, nystagmus, confusion, and even seizures—rather than diminished reflexes. Hyporeflexia would be atypical in this context.
Question 84
Severe lithium toxicity is MOST likely associated with:
A) Coarse tremor, confusion, and GI upset
B) Bradycardia, hypotension, and seizures
C) Hyperreflexia, muscle rigidity, and hyperthermia
D) Ataxia, nystagmus, and slurred speech
E) Polyuria, polydipsia, and renal failure
Answer: A. Coarse tremor, confusion, and GI upset
Explanation:
Toxic levels (>1.5 mmol/L) commonly produce gastrointestinal symptoms (nausea, vomiting, diarrhea) alongside central nervous system effects such as coarse tremor, confusion, and at higher levels, seizures and coma.
Question 85
Which medication is MOST likely to potentiate the therapeutic effects of electroconvulsive therapy (ECT) by lowering the seizure threshold?
A) Lithium
B) Clozapine
C) Lamotrigine
D) Escitalopram
E) Valproate
Answer: B. Clozapine
Explanation:
Clozapine has the strongest proconvulsant properties of the antipsychotics, lowering the seizure threshold and thereby enhancing seizure duration during ECT. Other agents like lithium and SSRIs have weaker effects.
Question 86
A 55-year-old man on clozapine 1,200 µ g/day develops generalized tonic‐clonic seizures. What is the NEXT step?
A) Stop clozapine immediately
B) Add valproate
C) Reduce clozapine dose
D) Start phenytoin
E) Monitor without intervention
Answer: B. Add valproate
Explanation:
Clozapine‐induced seizures are best managed by adding valproate as anticonvulsant prophylaxis. Dose reduction is considered only if seizures persist despite prophylaxis.
Question 87
Which antidepressant should be avoided with tamoxifen in a breast cancer survivor?
A) Amitriptyline
B) Fluoxetine
C) Sertraline
D) Mirtazapine
E) Venlafaxine
Answer: B. Fluoxetine
Explanation:
Fluoxetine is a potent CYP2D6 inhibitor that markedly reduces the conversion of tamoxifen to its active metabolite, compromising its efficacy. Agents with minimal CYP2D6 inhibition (e.g., venlafaxine, mirtazapine) are safer choices.
Question 88
Antipsychotic choice for an 80-year-old with eGFR >25 mL/min:
A. Olanzapine
B. Haloperidol
C. Amisulpride
D. Sulpiride
E. Quetiapine
Answer: A. Olanzapine
Explanation:
Amisulpride and sulpiride are renally excreted and therefore contraindicated in patients with significantly reduced renal function, whereas olanzapine and haloperidol (with appropriate dose adjustment) are safer options. Among second‐generation antipsychotics, olanzapine (5 mg/day) is recommended as first choice in renal impairment due to its hepatic metabolism and minimal renal excretion.
Question 89
A 72-year-old man with Lewy Body Dementia develops rigidity and confusion after starting risperidone. What is the NEXT step?
A) Continue risperidone
B) Switch to quetiapine
C) Add benztropine
D) Discontinue antipsychotics
E) Initiate electroconvulsive therapy
Answer: D. Discontinue antipsychotics
Explanation:
Patients with Lewy Body Dementia exhibit marked neuroleptic sensitivity, with antipsychotics—particularly risperidone and haloperidol—precipitating severe parkinsonism and confusion. Guidelines advise avoiding or withdrawing antipsychotics in this population to prevent adverse outcomes.
Question 90
A patient with chronic alcohol use presents with ataxia, slurred speech, and nystagmus. Which receptor system is MOST implicated in their neurological symptoms?
A. GABA-A and NMDA
B. Dopamine D2
C. Serotonin 5-HT2A
D. Acetylcholine
E. Glutamate AMPA
Answer: A. GABA-A and NMDA
Explanation:
Alcohol enhances inhibitory GABA-A receptor activity and antagonizes excitatory NMDA receptors. This combined effect underlies the CNS depression and cerebellar dysfunction—manifesting as ataxia, dysarthria, and nystagmus—seen in acute alcohol intoxication and withdrawal.
MRCPsych Paper B - 21 May 2025
MRCPsych Paper B - 21 May 2025
Question 1
Dundrum Crisis Scale primarily assesses:
A) Suicidal intent
B) Risk of violence
C) Functional impairment
D) Insight
E) Medication adherence
Answer: A. Suicidal intent
Explanation:
This scale helps evaluate the immediate danger of self-harm in patients in a psychiatric setting when assessing Suicidal intent.
Question 10
In evidence-based hierarchy, which study design is ranked HIGHEST?
A) Case-control
B) Cohort
C) Cross-sectional
D) Randomized controlled trial (RCT)
E) Systematic review
Answer: E. Systematic review
Explanation:
Systematic reviews synthesize results from multiple RCTs or observational studies, applying rigorous methods to minimize bias. Because they integrate all high-quality evidence on a question, they sit at the top of the evidence pyramid. Individual RCTs come next, then cohort and case-control studies, with cross-sectional near the bottom.
Question 11
A meta-analysis assumes fixed effects when:
A) Study heterogeneity is high
B) Effect sizes vary significantly
C) Population and interventions are homogeneous
D) Publication bias is present
E) Sample sizes are small
Answer: C. Population and interventions are homogeneous
Explanation:
A fixed-effect model treats all studies as estimating the same true effect size under identical conditions. That assumption only makes sense when subjects, interventions, and outcomes are sufficiently similar across studies—i.e., minimal heterogeneity. Thus, it's when Population and interventions are homogeneous.
Question 12
Which measure of central tendency is MOST appropriate for skewed data (e.g., income distribution)?
A) Mean
B) Median
C) Mode
D) Range
E) Standard deviation
Answer: B. Median
Explanation:
The Median marks the 50th percentile and isn’t influenced by extreme values. In skewed distributions—where a few very large or small observations would drag the mean away from the center—the median remains the best single-value summary of the “typical” case.
Question 13
Negative skew in a dataset is indicated by:
A) Mean > Mode
B) Mean < Mode
C) Median = Mean
D) Long right tail
E) Symmetrical distribution
Answer: B. Mean < Mode
Explanation:
Negative skew means the left tail extends farther than the right. That pulls the arithmetic mean below the mode (hence Mean < Mode), since extreme low values drag down the average more than the most common value.
Question 14
A physician decides to work at a rural clinic instead of a city hospital. This decision BEST illustrates:
A) Sunk cost fallacy
B) Opportunity cost
C) Marginal utility
D) Diminishing returns
E) Cost-benefit analysis
Answer: B. Opportunity cost
Explanation:
Opportunity cost is the value of the next-best alternative you give up. By choosing the rural clinic, the physician sacrifices whatever salary, caseload, or career development they could have had in the city. Recognizing that forgone benefit is exactly the concept of Opportunity cost.
Question 15
A researcher validates a new depression screening tool against the PHQ-9. Which type of validity are they assessing?
A) Predictive validity
B) Concurrent validity
C) Content validity
D) Construct validity
E) Criterion validity
Answer: B. Concurrent validity
Explanation:
Concurrent validity checks how well a new measure correlates with an established “gold standard” when both are administered at the same time. Here, comparing the new tool’s scores directly against PHQ-9 results in the same sample assesses Concurrent validity.
Question 2
A clinician adjusts the CAGE cut-off score from 2 to 3. What is the most likely effect on its diagnostic metrics?
A) Increased sensitivity, decreased specificity
B) Decreased sensitivity, increased specificity
C) Increased positive predictive value, unchanged negative predictive value
Answer: B. Decreased sensitivity, increased specificity
Explanation:
The CAGE questionnaire gives one point for each “yes” answer across its four items. By raising the threshold from two affirmative responses to three, you make it harder to screen positive. As a result, fewer true cases of alcohol misuse will be flagged (sensitivity falls), but you also reduce the number of false positives (specificity rises). In other words, you miss more people who do have a problem but you’re less likely to wrongly label someone as disordered. This results in Decreased sensitivity, increased specificity.
Question 3
A researcher conducts a study to evaluate a new depression screening tool. They report a P-value of 0.04. Which statement accurately interprets this result?
A) There is a 4% probability the null hypothesis is true.
B) The tool’s diagnostic accuracy is 96% reliable.
C) There is a 4% chance of falsely rejecting the null hypothesis.
D) The effect size is 0.04.
E) The study has 96% power.
Answer: C. There is a 4% chance of falsely rejecting the null hypothesis.
Explanation:
A P-value of 0.04 means that if in reality there were no effect (the null hypothesis is true), you would obtain data at least as extreme as observed in only 4 out of 100 repeated experiments. It does not tell you the probability that the null hypothesis itself is true, nor does it speak to accuracy, effect size, or power directly. Therefore, there is a 4% chance of falsely rejecting the null hypothesis (Type I error rate if the null is true).
Question 4
A study finds no difference in depression scores between two therapies but reports significant improvements in sleep as a secondary outcome. This MOST likely indicates:
A) Reporting bias
B) Confounding bias
C) Selection bias
D) Observer bias
E) Random error
Answer: A. Reporting bias
Explanation:
When researchers highlight favorable secondary outcomes (like sleep) despite a negative primary endpoint (depression scores), they risk giving a skewed picture of benefit. This selective emphasis—reporting only the “good news”—is classic Reporting bias. Other biases (confounding, selection, observer) involve design or measurement flaws, not selective presentation of results.
Question 5
In optimizing the CAGE questionnaire’s likelihood ratio for detecting harmful alcohol use, which cutoff score provides the highest positive likelihood ratio (LR+)?
A) 0 (any affirmative response)
B) 1
C) 2
D) 3
E) 4
Answer: C. 2
Explanation:
The positive likelihood ratio (LR+) equals sensitivity ÷ (1 – specificity). At a cutoff of two “yes” answers, the CAGE typically yields about 72% sensitivity and 75% specificity, giving an LR+ around 2.9. Lower cutoffs boost sensitivity but harm specificity, reducing LR+. Higher cutoffs improve specificity further but sacrifice too much sensitivity, again lowering LR+. Thus, a cutoff of 2 strikes the best overall balance for ruling in harmful use.
Question 6
Which statistical graph is used in meta-analyses to visually assess heterogeneity across studies?
A) Forest plot
B) Galbraith plot
C) Funnel plot
D) Scatterplot
E) Box plot
Answer: B. Galbraith plot
Explanation:
A Galbraith plot (also called a radial plot) graphs each study’s standardized effect size against its precision (the inverse of its standard error). Outlier studies will stand apart from the main cluster, flagging heterogeneity. While Forest plots summarize individual results and Funnel plots detect publication bias, only the Galbraith plot directly highlights between-study variability.
Question 7
What is the PRIMARY purpose of a Galbraith plot in systematic reviews?
A) Compare treatment effects
B) Assess publication bias
C) Evaluate heterogeneity
D) Visualize publication dates
E) Calculate odds ratios
Answer: C. Evaluate heterogeneity
Explanation:
Galbraith plots are specifically designed to flag heterogeneity by showing which studies deviate most from the average effect size once you adjust for precision. They help reviewers decide whether a fixed-effect or random-effects model is more appropriate, thus their primary purpose is to Evaluate heterogeneity.
Question 8
A patient answers “No” to 3 CAGE questions. What is the BEST interpretation?
A) High specificity for alcohol dependence
B) Moderate sensitivity for hazardous drinking
C) Low probability of alcohol use disorder
D) Confirmatory evidence of sobriety
E) Indication for liver function testing
Answer: C. Low probability of alcohol use disorder
Explanation:
A total score of zero on the CAGE (three “No” answers plus one remaining, assuming the question implies only one "Yes" or zero "Yes" answers if 3 are "No") yields high specificity—meaning few healthy people will test positive—and so a negative result effectively rules out disorder. It doesn’t absolutely confirm sobriety, but it does make alcohol use disorder unlikely, indicating a Low probability of alcohol use disorder.
Question 17
A study compares pain levels in three groups: medication A, B, and C. Which statistical test is MOST appropriate?
A) Unpaired t-test
B) Mann-Whitney U test
C) Kruskal-Wallis test
D) ANOVA
E) Chi-square test
Answer: C. Kruskal-Wallis test
Explanation:
The Kruskal-Wallis test is the non-parametric equivalent of one-way ANOVA and is used to compare three or more independent groups when the outcome (e.g., pain scores) may not follow a normal distribution. It ranks all observations and tests whether the distributions of ranks differ across groups, making it ideal for skewed or ordinal data without assuming equal variances or normality.
Question 18
A researcher compares baseline characteristics between treatment and control groups. Which test adjusts for confounding variables like age and sex?
A) Unpaired t-test
B) ANCOVA
C) Chi-square test
D) Fisher’s exact test
E) Wilcoxon signed-rank test
Answer: B. ANCOVA
Explanation:
Analysis of covariance (ANCOVA) combines ANOVA and regression: it compares means of a continuous outcome across groups while statistically controlling for one or more continuous or categorical covariates (e.g., age, sex). By including confounders in the model, ANCOVA isolates the effect of the main group factor on the outcome, reducing bias from imbalanced baseline characteristics.
Question 19
A psychiatrist compares pre- and post-intervention depression scores in two independent groups. Which test is BEST?
A) Paired t-test
B) Unpaired t-test
C) Mann-Whitney U test
D) Wilcoxon signed-rank test
E) Repeated-measures ANOVA
Answer: B. Unpaired t-test
Explanation:
When comparing the means of a continuous outcome between two independent groups at a single time point (e.g., post-intervention scores, or change scores if pre-subtracted), the two-sample (Unpaired t-test) is appropriate, provided the data are approximately normally distributed and variances are similar. It tests whether the difference in group means is statistically significant.
Question 20
A 2×2 contingency table shows a rare outcome (<5% frequency). Which test is MOST appropriate?
A) Chi-square test
B) Fisher’s exact test
C) McNemar’s test
D) Cochran’s Q test
E) Mantel-Haenszel test
Answer: B. Fisher’s exact test
Explanation:
In a 2×2 contingency table, if the expected frequency in any cell is less than 5 (common when outcomes are rare), the chi-square approximation becomes unreliable. Fisher’s exact test computes the exact probability of observing the data under the null hypothesis and is the standard method for small sample or low-frequency tables.
Question 21
A table reports adjusted odds ratio (OR = 1.5) and unadjusted OR = 2.0. Why is the adjusted OR lower?
A) Confounding by age
B) Smaller sample size
C) Type I error
D) Measurement bias
E) Random variation
Answer: A. Confounding by age
Explanation:
An unadjusted odds ratio may overestimate an association if a confounding variable (e.g., age) is unevenly distributed between groups and is associated with both the exposure and outcome. By including age as a covariate in a multivariable logistic regression model, the adjusted OR accounts for its effect, often attenuating the estimate when age was positively associated with both exposure and outcome. This is described as Confounding by age.
Question 27
Which study design best examines the association between lithium levels and depression severity?
A) Case-control
B) Cohort
C) Ecological
D) Cross-sectional
E) Meta-analysis
Answer: B. Cohort
Explanation:
A prospective cohort study enrolls participants based on exposure status (e.g., different lithium levels) and follows them over time to measure subsequent outcomes (depression severity). This design establishes temporal sequence, reduces recall bias, and allows direct measurement of incidence and risk related to the exposure. Thus, a Cohort study is most appropriate.
Question 28
Which study design is initially most appropriate to evaluate side effects of a new drug in 3 exposed individuals?
A) Case series
B) Case-control
C) N-of-1 trial
D) Cohort
E) Randomized controlled trial
Answer: A. Case series
Explanation:
A Case series describes and analyzes the clinical course of a small group of patients who share a specific exposure (here, 3 individuals on the new drug). It is a descriptive study useful for early identification of potential side effects before larger, more formal studies are undertaken.
Question 32
Two radiologists interpreting MRI results are assessing:
A. Concurrent validity
B. Content validity
C. Inter-rater reliability
D. Predictive validity
E. Internal consistency
Answer: C. Inter-rater reliability
Explanation (quick-revision points):
• Reliability deals with consistency; inter-rater reliability is consistency between observers rating the same cases.
• In psychometrics it is commonly quantified with κ (kappa) for categorical ratings or the intraclass correlation coefficient for continuous data.
A suicide risk tool’s scores correlate with future suicide attempts. This demonstrates:
A. Predictive validity
B. Concurrent validity
C. Content validity
D. Construct validity
E. Face validity
Answer: A. Predictive validity
Explanation:
• Predictive validity asks: does today’s score forecast a later, clinically meaningful outcome?
• Criterion validity has two flavours – concurrent (same time-point) and predictive (future event).
• High scores that accurately anticipate later self-harm indicate sound predictive validity.
Question 34
An ROC curve with an AUC of 0.86 and sensitivity of 100 % indicates:
A) Poor specificity
B) Optimal screening accuracy
C) High false-positive rate
D) Inadequate discriminative ability
E) Superior positive predictive value
Answer: B. Optimal screening accuracy
Explanation:
• AUC > 0.8 = excellent overall discrimination; a value of 0.86 is well inside this zone.
• Sensitivity 100 % means no diseased cases are missed. Where the curve hugs the top-left corner the test achieves its best balance.
• Specificity can still vary at different cut-offs, but the combination of very high sensitivity and high AUC reflects near-ideal screening performance, indicating Optimal screening accuracy.
Question 35
A researcher surveys 500 adults about their diet and exercise habits on a single day. This is an example of which study type?
A) Longitudinal cohort
B) Cross-sectional
C) Case-control
D) Ecological
E) Quasi-experimental
Answer: B. Cross-sectional
Explanation:
• Cross-sectional designs collect exposure and outcome data simultaneously at one point in time, yielding prevalence—not incidence.
• They are efficient, inexpensive, but cannot establish temporal direction or causality.
Question 36
To evaluate the impact of parental neglect on childhood depression prospectively while minimising recall bias, which design is best?
A) Cross-sectional survey
B) Retrospective case-control study
C) Prospective cohort study
D) Randomized controlled trial
E) Ecological study
Answer: C. Prospective cohort study
Explanation:
• A cohort recruits children before depression develops, classifies them by exposure (neglect) and observes future onset.
• This preserves temporal order and avoids reliance on retrospective memory.
Question 37
A study compares ECT use in patients with and without a history of stroke; expected cell sizes are < 5. Which test is most suitable?
A. Chi-square test
B. Fisher’s exact test
C. t-test
D. Mann-Whitney U test
E. ANOVA
Answer: B. Fisher’s exact test
Explanation:
• Fisher’s exact is the exact alternative to χ² when any expected frequency in a 2 × 2 table falls below 5.
• It calculates the precise probability of the observed distribution, avoiding χ²’s large-sample approximation.
Question 38
A clinician raises the CAGE threshold from ≥ 2 to ≥ 3 “yes” responses. Primary effect?
A. ↑ Sensitivity, ↓ Specificity
B. ↓ Sensitivity, ↑ Specificity
C. ↑ PPV, unchanged sensitivity
D. ↓ NPV, unchanged specificity
E. Unchanged sensitivity, ↑ Specificity
Answer: B. Decreased sensitivity, increased specificity
Explanation:
• Moving the cut-off to a stricter level produces fewer false positives (better specificity) but misses more true cases (lower sensitivity).
• The shift reflects the classic sensitivity-specificity trade-off illustrated in diagnostic test theory.
Question 39
In a 10 % prevalence setting, a test with 95 % sensitivity and 5 % false-positive rate (specificity = 95 %) yields what post-test probability after a positive?
• LR⁻ closer to 0 provides stronger ability to rule-out; 0.5 offers only modest exclusion.
Question 44
A study’s ROC plots specificity (not 1 − specificity) on the x-axis; AUC = 0.8. This indicates:
A. High diagnostic accuracy
B. Low sensitivity
C. Poor specificity
D. Inadequate discriminative ability
E. Need for more data
Answer: A. High diagnostic accuracy
Explanation:
• The numeric AUC is invariant to whether the horizontal axis is specificity or its complement; 0.8 still reflects strong discrimination, indicating High diagnostic accuracy.
Question 46
Which guideline ensures transparent reporting of sensitivity/specificity in diagnostic studies?
A. CONSORT
B. PRISMA
C. STARD
D. MOOSE
E. TREND
Answer: C. STARD
Explanation:
• STARD (Standards for Reporting Diagnostic Accuracy Studies) provides a 30-item checklist and flow-diagram template for clear reporting of test accuracy—including sensitivity, specificity and participant flow.
Question 47
The probability that a patient with a negative test truly lacks the disease is:
A) Sensitivity
B) Specificity
C) PPV
D) NPV
E) Likelihood ratio
Answer: D) Negative predictive value (NPV)
Explanation:
• NPV = True Negatives / (All Negative Results).
• Clinically, it reassures both patient and clinician that a negative result likely means absence of disease.
Question 48
A forest plot shows a pooled effect size with a narrow confidence interval crossing the null. What does this suggest?
A) High heterogeneity among studies
B) Statistically insignificant result
C) Strong evidence for efficacy
D) Publication bias
E) Large sample size
Answer: B) Statistically insignificant result
Explanation:
When the 95% confidence interval spans the null value (e.g., zero for mean differences), the result fails to reach statistical significance despite precision.
Question 49
A study compares two treatments for depression. Group A (n=214) receives mirtazapine, with 46/214 dropouts due to side effects; Group B (n=217) receives placebo, with 9/217 dropouts. What is the Number Needed to Harm (NNH)?
A) 5
B) 6
C) 7
D) 8
E) 9
Answer: B) 6
Explanation:
Calculate absolute risk increase: (46/214) – (9/217) ≈ 0.1735. The NNH is the reciprocal of this increase, 1/0.1735 ≈ 5.76, which rounds to 6. This aligns with the principle that NNH is the inverse of the adverse event difference.
Question 50
A depressed inpatient scores highly on both Scale A (100-item) and Scale B (12-item) with Spearman's ρ = 0.8 (p < 0.01). Which conclusion is MOST valid?
A. Scale B is a reliable replacement for Scale A
B. Both scales show excellent inter-rater reliability
C. A and B are highly linearly correlated
D. Scale A is more valid than Scale B
E. No conclusion can be drawn about validity
Answer: C. A and B are highly linearly correlated
Explanation:
Spearman’s rho measures the strength and direction of a monotonic (here near-linear) association between two instruments, indicating they assess a similar construct.
Question 51
A researcher compares mean cognitive scores across three groups (healthy controls, mild cognitive impairment, and Alzheimer's disease). Which statistical test is MOST appropriate?
A. Wilcoxon signed-rank test
B. ANOVA
C. Chi-square test
D. Mann-Whitney U test
E. Kruskal-Wallis test
Answer: B. ANOVA
Explanation:
One-way ANOVA is designed to compare the means of three or more independent groups when the outcome is continuous and approximately normally distributed.
Question 52
A study evaluates pain relief in 15 patients before and after acupuncture. Which test is MOST suitable?
A. Paired t-test
B. Wilcoxon signed-rank (rank sum) test
C. Chi-square test
D. Student's t-test
E. Regression analysis
Answer: B. Wilcoxon signed-rank (rank sum) test
Explanation:
For paired, nonparametric data (pain scores before vs. after in the same individuals), the Wilcoxon signed-rank (rank sum) test compares median changes without assuming normal distribution.
Question 53
A psychiatrist compares smoking rates (smokers vs. non-smokers) in patients with schizophrenia vs. controls. Which test is MOST appropriate?
A. Chi-square test
B. ANOVA
C. Mann-Whitney U test
D. t-test
E. Regression analysis
Answer: A. Chi-square test
Explanation:
The χ² test assesses associations between two categorical variables (smoking status and diagnostic group) by comparing observed vs. expected frequencies.
Question 54
Which statistical test compares means across three independent ordinal groups (e.g., consultants, nurses, patients)?
A) Paired t-test
B) One-way ANOVA
C) Kruskal-Wallis test
D) Chi-square test
E) Logistic regression
Answer: C) Kruskal-Wallis test
Explanation:
When comparing an ordinal outcome across three or more independent groups without assuming normality, Kruskal–Wallis provides a nonparametric alternative to ANOVA.
Question 55
A researcher compares baseline Hamilton Depression Rating Scale scores across three timepoints. Which statistical method is MOST suitable?
A) Repeated-measures ANOVA
B) Mann-Whitney U test
C) Pearson correlation
D) Chi-square test
E) Linear regression
Answer: A) Repeated-measures ANOVA
Explanation:
Repeated-measures ANOVA analyzes within-subject changes over multiple timepoints, accounting for the correlation between repeated observations.
Question 56
Which statistical test is MOST appropriate for analyzing data that follows a normal distribution?
A) Mann-Whitney U test
B) Kruskal-Wallis test
C) Parametric ANOVA
D) Spearman’s rank correlation
E) Wilcoxon signed-rank test
Answer: C) Parametric ANOVA
Explanation:
One-way ANOVA assumes normally distributed residuals and homogeneity of variances, and is the standard method for comparing means across three or more independent groups when these assumptions hold.
Question 57
A study compares mortality rates between two intensive care units using a dichotomous outcome (survived/deceased). Which statistical test is MOST appropriate?
A) Paired t-test
B) Mann-Whitney U test
C) Chi-square test
D) Pearson correlation
E) ANOVA
Answer: C) Chi-square test
Explanation:
For two independent groups and a binary outcome, the chi-square test of independence assesses whether the proportion of deaths differs significantly between units by comparing observed vs. expected frequencies.
Question 58
A researcher compares mean blood glucose levels between three groups (healthy controls, prediabetes, and diabetes). Which statistical test is MOST appropriate?
A) Wilcoxon signed-rank test
B) ANOVA
C) Chi-square test
D) Mann-Whitney U test
E) Kruskal-Wallis test
Answer: B) ANOVA
Explanation:
Comparing the means of a continuous variable across three independent groups requires a one-way ANOVA under normality assumptions. Non-parametric alternatives are reserved for skewed distributions or ordinal data.
Question 59
A study evaluates pain relief in 20 patients before and after acupuncture. Which test is MOST suitable?
A) Paired t-test
B) Wilcoxon signed-rank test
C) Chi-square test
D) ANOVA
E) Student’s t-test
Answer: B) Wilcoxon signed-rank test
Explanation:
For paired (before/after) measurements that may not be normally distributed, the Wilcoxon signed-rank test compares median changes without relying on normality.
Question 60
A study of 1,054 schizophrenia patients categorizes them into three groups, including suicide attempters. Which statistical test is MOST appropriate to evaluate suicide risk factors?
A) Chi-square test
B) t-test
C) ANOVA
D) Logistic regression
E) Cox proportional hazards model
Answer: A) Chi-square test
Explanation:
To examine associations between categorical risk factors and group membership (including suicide attempters), the chi-square test compares observed vs. expected counts across categories.
Question 61
A trial compares CBT and DBT for binge-eating disorder. Which statistical test is most appropriate to compare remission rates (non-normal distribution)?
A) Mann-Whitney U test
B) Independent t-test
C) Chi-square test
D) ANOVA
E) Pearson correlation
Answer: A) Mann-Whitney U test
Explanation:
When remission rates are treated as non-parametric continuous or ordinal data between two independent groups, the Mann-Whitney U test compares their distributions without assuming normality.
Question 62
Which statistical test is MOST appropriate for analyzing the relationship between two ordinal variables?
A) Pearson’s correlation
B) Spearman’s rank correlation
C) Student’s t-test
D) ANOVA
E) Chi-square test
Answer: B) Spearman’s rank correlation
Explanation:
Spearman’s ρ assesses monotonic associations between variables measured on an ordinal scale or when data are not normally distributed, ranking each variable before computing correlation.
Question 63
Which statistical test is used to compare proportions between two categorical groups (e.g., treatment success vs. failure)?
A) Paired t-test
B) Mann-Whitney U test
C) Chi-square test
D) ANOVA
E) Pearson correlation
Answer: C) Chi-square test
Explanation:
For two independent categorical variables and binary outcomes, the chi-square test evaluates differences in proportions by comparing observed and expected frequencies.
Question 64
Three electroconvulsive therapy (ECT) modalities (bilateral, unilateral, ultrabrief) are compared for treatment-resistant depression. The outcome is remission rates. Which statistical test is MOST suitable?
A) Paired t-test
B) Mann-Whitney U test
C) Chi-square test
D) ANOVA
E) Pearson correlation
Answer: C) Chi-square test
Explanation:
With three independent groups and a binary (remission yes/no) outcome, the chi-square test of independence assesses whether remission proportions differ across modalities.
Question 65
A patient with chronic alcohol use presents with ataxia, slurred speech, and nystagmus. Which receptor system is MOST implicated in their neurological symptoms?
A) GABA-A and NMDA
B) Dopamine D2
C) Serotonin 5-HT2A
D) Acetylcholine
E) Glutamate AMPA
Answer: A) GABA-A and NMDA
Explanation:
Alcohol enhances inhibitory GABA-A receptor function and antagonizes NMDA-type glutamate receptors, leading to cerebellar dysfunction manifesting as ataxia, dysarthria, and nystagmus.
Question 66
Which statistical test is MOST appropriate to compare categorical data (ECT vs. stroke occurrence) between two independent groups?
A) Pearson’s chi-square
B) Student’s t-test
C) Mann-Whitney U test
D) ANOVA
E) Fisher’s exact test
Answer: A) Pearson’s chi-square
Explanation:
With sufficient cell counts (>5), the chi-square test assesses the association between two categorical variables. If any expected cell counts fell below 5, Fisher’s exact test would be used instead.
Question 67
Which statistical test does NOT assume normal data distribution?
A) Student’s t-test
B) Pearson’s correlation
C) Mann-Whitney U test
D) ANOVA
E) Linear regression
Answer: C) Mann-Whitney U test
Explanation:
The Mann-Whitney U test is a non-parametric alternative to the unpaired t-test, comparing two independent groups without requiring normality or equal variances.